Download Guide Questions

Document related concepts

Prenatal testing wikipedia , lookup

Medical ethics wikipedia , lookup

Adherence (medicine) wikipedia , lookup

Differential diagnosis wikipedia , lookup

Patient safety wikipedia , lookup

Electronic prescribing wikipedia , lookup

Dysprosody wikipedia , lookup

Patient advocacy wikipedia , lookup

List of medical mnemonics wikipedia , lookup

Transcript
SMALL GROUP DISCUSSIONS
STUDENT’S STUDY GUIDE
I.
Date and Time: To be announced
II.
Facilitators: Faculty of the Department of Pediatrics
III.
Form of Teaching: Small Group Discussion
IV.
Prerequisites:
The students should have passed all courses in Learning Unit III and Learning Unit IV
particularly those dealing with normal growth and development, and pathophysiology
of diseases specifically those commonly seen in infancy, childhood, and adolescence.
V.
Overview of the Activity:
There will be five common clinical ambulatory symptoms to be discussed in each of the
five small group sessions per rotation. The block of 20 students will be divided into four
small groups. Two differential diagnoses of a clinical symptom will be discussed, each
by a group of five students, during each session. The whole block will be given all cases
to be discussed at the start of each rotation. Those not presenting will act as reactors to
the presenting groups. Reactions may be in the form of questions, comments, or
additional information pertinent to the discussion. One consultant will facilitate the
discussion of the two cases per session.
These sessions are part of a 4-week module dealing with the principles of diagnosis,
management and preventive care of common ambulatory conditions in pediatrics.
VI.
Course Outcomes and Content:
At the end of each session, the student should be able to:
1.
Define the symptom.
2.
Discuss the pathophysiology of the symptom.
3.
Enumerate and justify common differential diagnoses based on the history and
physical examination.
4.
Identify and interpret pertinent laboratory work-up necessary to come up with the
final diagnosis.
5.
Give a sound and rational diagnosis based on history, physical examination, and
laboratory parameters.
6.
Discuss the pathophysiology of the final diagnosis.
7.
Discuss the initial management of the final diagnosis.
8.
Cite and discuss appropriate clinical practice guidelines on the diagnosis and
management of the final diagnosis.
The following are the symptoms and specific problems which will be presented during
the entire school year. Not all will be covered per rotation but will be included during
the final examination.
1. Cough
a. Acute Respiratory Tract
Infection
b. Bronchial
Asthma/
Allergic Rhinitis
c. Pulmonary Tuberculosis
d. Gastroesophageal Reflux
2. Diarrhea
a. Viral
Acute Gastroenteritis
b. Bacterial
Acute Gastroenteritis
c. Non-infectious Diarrhea
d. Cow’s Milk Allergy
3. Fever
a. Dengue
Hemorrhagic
Fever
b. Typhoid Fever
c. Systemic Viral Illness
d. Urinary Tract Infection
4. Pallor
a.
b.
c.
d.
Iron-deficiency Anemia
Hemolytic Anemia
Hematologic Malignancy
Chronic Renal Disease
5. Edema
a. Renal Failure
b. Congestive Heart Failure
c. Liver Failure
d. Malnutrition
6. Failure to Thrive/Short Stature
a. Malnutrition
b. Congestive Heart Failure
c. Chronic Renal Disease
d. Endocrine Disorder
7. Seizures
a. Epilepsy
b. Meningitis
c. Tumor
d. Metabolic
8. Developmental Delay
a. Cerebral Palsy
b. Hypothyroidism
c. Autism
d. Congenital
CNS Infections
9. Jaundice
a. Hepatitis
b. Biliary Atresia
c. Physiologic Jaundice
d. Hypothyroidism
10. Headache
a. Migraine
b. Brain tumor
11. Abdominal pain
a. Constipation
b. Functional
Abdominal Pain
12. Musculoskeletal pain
a. Rheumatic Fever
b. Well Child
VII.
Following are guide questions for the clinical symptoms mentioned above (More
detailed guides will be provided for the cases to be discussed in each block).
A. COUGH
1. Define cough.
2. What is the pathophysiology and mechanism of cough?
3. What are the diseases which may present as acute cough in children? As
chronic cough?
4. How will you differentiate the following disease entities based on the
character and timing of cough, history, and physical examination?
a. Acute Respiratory Tract Infection
b. Bronchial Asthma /Allergic Rhinitis
c. Pulmonary Tuberculosis
d. Gastroesophageal Reflux
5. What are the diagnostic work-up necessary to narrow down your
differential diagnoses and confirm your final diagnosis?
6. What are the other pertinent data necessary in initiating treatment for the
above-mentioned diseases?
7. Are there clinical guidelines which may help you in the diagnostic and
therapeutic management of these diseases?
8. What are the preventive aspects in the management of common diseases
presenting as cough?
B. DIARRHEA
1. Define diarrhea.
2. What are the different processes involved in the pathophysiology of
diarrhea?
3. What are the common causes of diarrhea in children?
4. How will you differentiate the following based on the character of stools,
history, and physical examination?
a. Viral Acute Gastroenteritis
b. Bacterial Acute Gastroenteritis
c. Non-infectious Diarrhea
d. Cow’s Milk Allergy
5. Is there any laboratory work-up necessary to differentiate the above
causes of diarrhea?
6. How will you prioritize your management of a child with diarrhea?
7. How will you assess and treat the degree of dehydration based on the
WHO Protocol on the Management and Prevention of Diarrheal
Diseases?
8. What are the preventive aspects in the management of diarrheal diseases?
C. FEVER
1. Define fever.
2. What is the pathophysiology of fever?
3. What are the different types of fever?
4. What are the common diseases in which fever is the presenting or
prominent complaint?
5. How will you differentiate the following diseases based on the character
of the fever, history, and physical examination?
a. Dengue Hemorrhagic Fever
b. Typhoid Fever
c. Systemic Viral Illness
d. Urinary Tract Infection
6. What are the diagnostic work-up necessary to narrow down your
differential diagnoses and confirm your final diagnosis?
7. What is the treatment for the above-mentioned diseases?
8. Are there clinical guidelines which may help you in the diagnostic and
therapeutic management of these diseases?
D. PALLOR/ANEMIA
1. What is pallor? Anemia?
2. What are the different mechanisms involved in the pathophysiology of
anemia?
3. What are the common causes of anemia in children?
4. What is physiologic anemia of infancy?
5. How do the following cause anemia in children?
a. Iron-deficiency Anemia
b. Hemolytic Anemia
c. Hematologic Malignancy
d. Chronic Renal Disease
6. How will you differentiate these conditions based on history and physical
examination?
7. What are the laboratory work-ups necessary to document the above
causes of anemia?
8. How will you approach the management of these disease entities?
E. EDEMA
1. What is edema?
2. What is the pathophysiology of edema? What is the Starling’s law of
ultrafiltration?
3. What organ systems may be involved in a child presenting with edema?
4. What are the disease entities in children which may cause edema through
the following mechanisms?
a. Increased (capillary) hydrostatic pressure
b. Decreased (capillary) oncotic pressure
c. Increased capillary permeability
d. Sodium and water retention
5. How will you differentiate the following causes of edema based on the
character of edema, history, and physical examination?
a. Renal Failure
b. Congestive Heart Failure
c. Liver Failure
d. Malnutrition
6. What is the pathophysiology of the above causes of edema?
7. What are the diagnostic work-ups that you will request for based on the
most likely cause of edema? What are the expected findings?
8. What are the common problems associated with the above-mentioned
disease entities? How will you manage each problem? Give the rationale
for the specified treatment.
F. FAILURE TO THRIVE/SHORT STATURE
1. What is failure to thrive?
2. How will the following mechanisms lead to failure to thrive or growth
deficiency? Cite examples of conditions presenting with failure to thrive
due to the following mechanisms.
a. Qualitative and quantitative inadequacy of food intake
b. Defects in food assimilation
c. Loss of food substances
d. Failure of utilization or increased metabolism
3. When do you consider a child to be understatured?
4. What is a normal variant or constitutional short stature?
5. How do you determine the percentile ranking of measurements of
physical growth?
6. How will you differentiate the following causes of failure to thrive/short
stature based on history and physical examination?
a. Malnutrition
b. Congestive Heart Failure
c. Chronic Renal Disease
d. Endocrine Disorder
7. What are the diagnostic work-ups necessary to narrow down your
differential diagnoses and confirm your final diagnosis?
8. What are the aspects of management of the above conditions?
G. SEIZURES
1. What is seizure? Epilepsy?
2. What is a benign febrile seizure?
3. What are the common causes of seizures in children?
4. How will you differentiate the following causes of seizures based on
history and physical examination?
a. Epilepsy
b. Meningitis
c. Tumor
d. Metabolic Causes
5. What are the diagnostic work-ups that you will request for to confirm
and further evaluate the above conditions?
6. What are the other pertinent data necessary in initiating treatment for the
above-mentioned diseases?
7. How will you manage the patient based on the etiology and classification
of seizure?
H. DEVELOPMENTAL DELAY/MENTAL RETARDATION
1. What is developmental delay? Mental retardation?
2. How will you assess developmental delay/ mental retardation in
children?
3. What are the potential contributing factors in the pathogenesis of mental
retardation?
4. What are the common causes of developmental disabilities in children?
5. How will you differentiate the following causes of developmental
delay/mental retardation based on history and physical examination?
a. Cerebral Palsy
b. Hypothyroidism
c. Autism
d. Congenital CNS Infections
6. What are the indications for specific laboratory tests in a child with
mental retardation?
7. What are the aspects of management of children with developmental
delay/mental retardation?
I. JAUNDICE
1. What is jaundice?
2. What is the pathophysiology of jaundice?
3. What is physiologic jaundice?
4. When is jaundice pathologic?
5. What is neonatal cholestasis?
6. What are the common causes of jaundice in children based on onset of
jaundice, type of hyperbilirubinemia, history, and physical examination?
7. How will you differentiate the following based on onset of jaundice, type
of hyperbilirubinemia, history, and physical examination?
a. Hepatitis
b. Biliary Atresia
c. Physiologic Jaundice
d. Hypothyroidism
8. What are the diagnostic work-ups necessary to confirm the etiology of
jaundice?
9. What are the aspects in the management of a child presenting with
jaundice?
J. HEADACHE
1. What are the most important causes of headache in children?
2. How will you differentiate the following causes of headache based on
character of headache, history, and physical examination?
a. Migraine
b. Increased Intracranial Pressure
c. Tension or Stress Headaches
3. What are the indications for neuroimaging a child with headaches?
4. What are the diagnostic and treatment modality of choice for each of the
above causes of headache?
K. ABDOMINAL PAIN
1. What are the possible organ systems involved in a child presenting with
abdominal pain?
2. What are the common conditions presenting with acute abdominal pain
in children? Chronic or recurrent abdominal pain?
3. How will you differentiate a surgical (“acute”) from a non-surgical
abdomen?
4. How will differentiate the cause of abdominal pain based on the nature
and location of pain-provoking lesion, history, and physical examination?
5. When is an abdominal pain functional or non-organic?
6. What are the laboratory work-ups necessary to narrow down your
differential diagnoses and confirm the cause of abdominal pain?
7. How will you manage the patient based on your final diagnosis?
L. MUSCULOSKELETAL PAIN
1. What are “growing” pains?
2. What is arthritis?
3. What are the common childhood diseases in which musculoskeletal/limb
pain is the presenting or prominent complaint? How will you
differentiate them based on the history and physical examination?
4. How will you approach the diagnosis and management of the following
causes of limb pain in children?
a. Rheumatic Fever
b. Growing pains
VIII.
References:
A. Nelson WE, Behrman RE, Kliegman RM, Arvin AM (eds). Nelson Textbook of
Pediatrics. 16th ed. W.B. Saunders Company, Philadelphia. 2000.
B. Morris G (ed). Pediatric Diagnosis: Interpretation of Symptoms and Signs in Children and
Adolescents. 6th ed. W.B. Saunders Company, Singapore. 1998.
C. Del Mundo F, Estrada FA, Santos Ocampo PD, Navarro X (eds). Textbook of Pediatrics
and Child Health. 4th ed. JMC Press, Quezon City, Philippines. 2000.
Note: Additional study guide questions and references will be provided for the specific
cases to be discussed during the small group sessions.
EDEMA
PEDIATRIC CASE NUMBER 1-1
HISTORY: A 9-year-old female was brought to the outpatient department for bipedal edema.
Two weeks ago, the patient was noted to have easy fatigability and fever for which no consult
was made and no medications taken. One week prior to consultation, she started to have
bipedal edema, body malaise, palpitations, slight difficulty of breathing, and 1-pillow
orthopnea. Persistence of symptoms prompted consult to our institution.
PHYSICAL EXAMINATION: HR 120/min., RR 28/min., T 37.6 C, pale, no neck vein
engorgement, in moderate cardio-respiratory distress, w/ bibasal rales, no precordial bulge,
Adynamic precordium, PMI/AB 5th ICS LMCL, muffled heart sounds, tachycardic, regular
rhythm, (+) grade 3/6 holosystolic murmur at the apex radiating to the axilla and back, no
hepatomegaly, full and equal pulses, (+) grade 2 bipedal edema.
Guide Questions:
1. What additional information in the history would you ask for and why?
2. What specific physical findings would you look for and why?
3. What are the features in the history and physical examination which suggest a cardiac
cause of edema?
4. What is the pathophysiology of Congestive heart failure (CHF)?
5. What is your working diagnosis? Differential diagnosis? Basis?
6. What initial laboratory tests would you request and why? What are your
expected findings?
X-RAY: Chest X-ray shows cardiomegaly with left atrial & ventricular enlargement, increased
PVMs & no infiltrates.
LABORATORY: CBC shows anemia and elevated WBC with PMN predominance. Serum
electrolytes are normal. ASO, ESR, & CRP are elevated. Throat culture is negative.
ELECTROCARDIOGRAM:
hypertrophy.
ECG shows sinus tachycardia, normal axis with no chamber
Guide Questions:
1. What is your working diagnosis now?
2. How do you confirm your diagnosis?
3. What would you do next?
4. What are the non-pharmacologic and pharmacologic management which you will give
this patient?
a. Identify and prioritize problem list.
b. How would you manage the identified problems in this patient? Give the
rationale and necessary precautions for your management.
PEDIATRIC CASE NUMBER 1-2
HISTORY: A 10-month old girl consulted at the emergency room for abdominal distension.
Her mother claims that the history started two weeks before admission when she started to
have intermittent febrile episodes associated with reappearance of jaundice, acholic stools and
gradual abdominal enlargement. The child was diagnosed to have extrahepatic biliary atresia at
two months and underwent corrective operation for this. Her jaundice resolved after the
surgery and she has been maintained on the following medications: phenobarbital,
multivitamins and vitamin K. Presently, she can stand with support.
PHYSICAL EXAMINATION: On physical examination, the patient was noted to have the
following findings: weight 8 kg, length 70 cm, BP 80/40, HR 100/min, RR 24/min, T 39C. She
was awake but irritable, with generalized jaundice. She had pink conjunctivae, icteric sclerae, no
nasal or aural discharge and no palpable cervical lymphadenoapathy. Her lung fields were
clear. Heart sounds were distinct, with the apex beat at the 5th ICS, left midclavicular line. The
abdomen was distended with an everted umbilicus, prominent periumbilical veins. Fluid wave
was elicited and the liver was hard and smooth 4 cm below the right costal margin, 4 cm below
the xiphoid, with the spleen palpable 7 cm below the left costal margin. Pulses were full, nail
beds pink, no clubbing, but palmar erythema was noted. There was good muscle tone.
LABORATORY: Laboratory exams showed hemoglobin 8.0 g/L (Normal 12-15 g/L); white
cell count 15.1 x 109/L (Normal 4-11 x 109/L), 70% polymorphs, 28% lymphocytes; platelet
count: 100 x 1012/L (Normal 150-450 x 1012/L); Prothrombin time International normalised ratio
of 1.8; albumin 18 g/L (Normal 35-45 g/L), Na 128 mmol/L (135-145 mmol/L); K 4.0 mmol/L
(Normal 3.5-5.5 mmol/L). Hepatobiliary ultrasound showed liver parenchymal disease.
Guide Questions:
1.
2.
3.
4.
What is the cause of ascites in this patient?
What are the other signs of end stage liver disease is this patient?
What is the most appropriate management for this patients?
Are there any other complications of ascites that may occur in this patient? If so, how
would you manage this?
5. What advice would you give to the mother as to the long term survival of this patient?
PEDIATRIC CASE NUMBER 1-3
HISTORY: An 8-month-old female infant was brought to the local health center because of
generalized edema. This problem started two months prior to consultation and was associated
with a decrease in activity and mouth and skin lesions.
NUTRITIONAL HISTORY: The nutritional history revealed that she was never breast fed.
From the age of four months, her infant formula was shifted to Bear Brand (filled milk),
prepared with two scoops of milk in 8 oz of “am.” Complementary feeding only consisted of
“lugaw” started at the age of six months.
PHYSICAL EXAMINATION: Physical examination showed an apathetic child, with a weight
of 9 kg and a body length of 66 cms. Her vital signs were as follows: HR=92/min; RR=26/min;
T=36.8°C. Diffuse areas of flaky hyperpigmentation were noted on the buttocks and extremities.
Her hair was light-colored and sparse in distribution. Periorbital edema with pale conjunctivae,
fullness of the cheeks, pale lips, and ulcers on the angle of the lips were noted. There were clear
breath sounds, regular cardiac rate and rhythm with no murmur. The abdomen was distended
with the presence of fluid wave. The liver edge was firm and was palpable about 4 cms below
the right costal margin. There was no splenomegaly. There was bipedal edema.
LABORATORY: Hemoglobin 6 g/dL; WBC 9.35 × 109/L with 45% polymorphs, 55%
lymphocytes; hypochromic, microcytic anemia with no toxic granules; serum albumin was 9
g/L.
Guide questions:
1. What other points in the history and physical examination do you wish to be included?
2. What is your clinical impression and basis for this impression?
3. What are the predisposing factors in the development of edema in this case?
4. What are your differential diagnoses?
5. What other laboratory examinations should you order?
6. Outline your plan of management for this patient.
PEDIATRIC CASE NUMBER 1-4
GENERAL DATA: J.M., a 10-year-old female from Dasmariñas, Cavite who was admitted for
the first time at PGH due to edema.
HISTORY OF PRESENT ILLNESS: Three weeks PTA, patient had fever, cough, and colds.
She was given paracemtamol and carbocisteine which afforded temporary relief. Three days
PTA, the patients was noted to have facial edema and abdominal distention. Patient
complained of decreased urine output. Mother also noted tea-colored urine. She was brought
to a private MD for consult, diagnosis was unknown to the mother and was given with
unrecalled medications. There was still no improvement of symptoms.
Persistence of edema prompted consult at PGH.
PHYSICAL EXAMINATION:
GENERAL: The patient was conscious, irritable, coherent, not in cardiopulmonary distress.
VITAL SIGNS: BP 150/90, HR 90/minute, RR 24/minute, weight 49 kg, height 152 cm, BSA 1.4.
HEENT: Anicteric sclerae, slightly pale conjunctiva, with periorbital and facial edema.
CHEST/LUNGS: Symmetrical chest expansion, harsh breath sounds.
HEART: Adynamic precordium, regular cardiac rate and rhythm, no murmur.
ABDOMEN: NABS, soft, no organomegaly, tenderness, or masses.
EXTREMITIES: Full pulses with grade II bipedal edema.
LABORATORY EXAMINATION:
CBC: hgb 83
hct .24
plt 388
Seg 0.80
lymph 0.18
ELECTROLYTES: Na 136 mmol/L (140-148 mmol/L)
K 3.7 mmol/L (3.6 – 5.2 mmol/L)
Alb 19 g/L (34 – 50 g/L)
phosphorus 1.65 mmol/L (0.81 – 1.58 mmol/L)
BUN 28.43 mmol/L (2.60 – 6.40 mmol/L)
Crea 893 mmol/L (53 – 115 umol/L)
Cholestrol 9.78 mmol/L (4.20 – 5.20 mmol/L)
24-HOUR URINE COLLECTION:
total volume
800 ml/24 hr (500 – 2000 ml/24 hr)
urine creatinine
0.70 g/24 hr (0.60 – 2.50 g/24 hr)
urine total protein
3.55 g/24 hr (0.00 – 1.100 g/24 hr)
ABG: pH 7.28
pCO2 30
pO2 135
HCO3 12.4
BE –12
URINALYSIS: yellow/hazy
Sp.gr. 1.010
sugar negative
pH 8.5
protein 4+
RBC innumerable
cast negative
WBC 1-4.HPF
Crystals negative
epithelial cells rare
Bacteria rare
mucus threads negative
sats 98.8%
KUB ULTRASOUND: Kidneys are enlarged with increase in parenchymal echogenicity and
fair corticomedullary differentiation.
The right kidney measures 13.2 cm x 6.6 cm x 4.6 cm.
The left kidney measures 14.4 cm x 7.3 cm x 8.7 cm
The central echo complexes are intact. No lithiasis seen.
Bilateral enlarged kidneys with parenchymal disease.
Guide Questions:
1. What is the most likely diagnosis?
2. How do you evaluate the tea-colored urine? Is it hematuria or pigmenturia?
3. Enumerate the different etiologies for glomerular hematuria? Non-glomerular?
4. What is the history, physical exam, and laboratory tests will support your diagnosis?
5. Give three common differentials for this case.
6. Correlate the criteria for diagnosis and principles of management with the
pathophysiologic mechanism of the disorder.
COUGH
PEDIATRIC CASE NUMBER 2-1
HISTORY: The patient is a 9-month-old female who had a 2-week history of cough and colds
with no medications taken or consults done. This morning, she had poor suck with moderate to
high-grade fever. Four hours PTC, she was noted to have difficulty of breathing and was
brought for consult.
PHYSICAL EXAMINAITON:
GENERAL: Awake, irritable, in mild respiratory distress and weighs 8 kg.
VITAL SIGNS : RR 60 per minute, HR 110, and Temperature 37.8°C.
HEENT: Anicteric sclerae, pinkish palpebral conjunctivae. No ear discharge, positive alar
flaring. Negative tonsillopharyngeal congestion. Trachea in midline, negative cervical
lymphadenopathy.
LUNGS: Symmetrical chest expansion, positive suprasternal & intercostals retractions, with
crackles over both lung fields.
HEART: Regular cardiac rhythm, no murmur.
ABDOMEN: Abdomen slightly globular, with normoactive bowel sounds, and soft.
EXTREMITIES: Extremities are grossly normal, full pulses, no cyanosis.
The ER consultant requested for a chest x-ray which showed infiltrates on both basal lung
fields.
Guide Questions:
1.
2.
3.
4.
5.
6.
What are the essential aspects of the history and PE?
What are the differential diagnoses for ARI?
What is the most likely diagnosis?
What is the etiologic agent?
What are the useful diagnostic tools for the evaluation and diagnosis of PCAP?
What is the management for the different ARI?
PEDIATRIC CASE NUMBER 2-2
HISTORY: A 3-year-old female was brought to the ER with difficulty of breathing. Two days
PTC, she developed moderate to high-grade fever and sore throat. The patient was given
paracetamol 120mg/5ml, 5 ml every 4 hours with only temporary relief. Few hours PTC, the
patient developed progressive difficulty of breathing and swallowing. She was also noted to
have drooling of saliva, hyperextension of neck and prefers sitting in a tripod position.
PHYSICAL EXAMINATION:
GENERAL: Patient is awake, febrile, positive inspiratory stridor, and in respiratory distress.
Weight 14 kg.
VITAL SIGNS : RR 50 per minute, HR 13 per minute, and Temperature is 39.6°C.
HEENT: Anicteric sclerae, pinkish palpebral conjunctiva, and positive alar flaring.
LUNGS: Symmetrical chest expansion, positive suprasternal & intercostals retractions, clear
breath sounds with poor air entry in both lung fields.
HEART: Adynamic precordium, regular cardiac rhythm, no murmur.
ABDOMEN: Abdomen is flat, soft, and non-tender.
EXTREMITIES: Extremities are grossly normal with full equal pulses and no cyanosis.
The pedia resident on duty requested for a neck x-ray AP soft tissue lateral which showed
“thumb sign.”
Guide Questions:
1.
2.
3.
4.
5.
6.
What are the essential aspects of the history and PE?
What are the differential diagnoses for ARI?
What is the most likely diagnosis?
What is the etiologic agent?
What are the useful diagnostic tools for the evaluation and diagnosis of PCAP?
What is the management for the different ARI?
PEDIATRIC CASE NUMBER 2-3
HISTORY: A 2-year-old male child was brought to the emergency room because of difficulty of
breathing. History revealed that the patient was having cough for 5 days, described as
occurring often at night and early morning, paroxysmal in character. There was no history of
fever. The patient is a diagnosed case of bronchial asthma since 1 year of age with poor asthma
control even if maintained on anti-asthma medications with good compliance. He also has
episodes of vomiting, noted to occur if patient would lie down immediately after eating. There
is no family history of bronchial asthma or atopy.
PHYSICAL EXAMINATION: The patient was in slight cardiopulmonary distress with
following findings: HR 100/min, RR 44/min, T 370C, weight 10 kg length 82 cm. There were
alar flaring, suprasternal and subcostal retractions, fair air entry with wheezing sound all over
lung fields. Chest x-ray revealed hyperaerated airways with presence of infiltrates on the right
upper lung field. Hemoglobin was 9.7 g/dL. WBC was 11 × 109/L with 66% percent
polymorphs and 16% lymphocytes.
The patient was placed on NPO and oxygen support given. Intravenous steroid and inhaled
bronchodilator were started. On the 2nd hospital day, feeding was started. Barium swallow done
on the 3rd hospital day revealed significant reflux to the proximal esophagus. Patient was
started on domperidone and famotidine.
Guide Questions
1. What is the pathophysiology and mechanism of cough in this patient?
2. How will you differentiate GERD from other diseases presenting with cough based on
the character and timing of cough, history, and physical examination?
3. What diagnostic work-ups are necessary to confirm your final diagnosis?
4. What are other pertinent data necessary for initiating treatment for this patient?
5. Are there clinical guidelines which may help you in the diagnostic and therapeutic
management of this patient?
6. What are the preventive aspects in the management of this patient?
PEDIATRIC CASE NUMBER 2-4
HISTORY: A 2-year-old female was brought to the ER for consult. Three days PTC, she
developed watery nasal discharge with occasional cough and low-grade fever. One day PTC,
she was noted to have hoarseness of voice with inspiratory stridor which was noted to be worse
during sleep and aggravated by crying.
PHYSICAL EXAMINATION:
GENERAL: Awake, afebrile, irritable, in mild respiratory distress. Weighs 12.5 kg.
VITAL SIGNS : HR130 per minute, RR 50, temperature 38.2°C.
HEENT: Anicteric sclerae, pinkish palpebral conjunctivae. Patient presents with watery nasal
discharge with slight alar flaring. Trachea in midline, positive cervical lymphadenopathy and
hyperemic pharyngeal walls.
LUNGS: Symmetrical chest expansion, positive suprasternal retractions, positive rhonchi.
HEART: Regular cardiac rhythm, no murmur.
ABDOMEN: Abdomen is flat and soft with noromoactive bowel sounds.
EXTREMITIES: Grossly normal, full equal pulses, no cyanosis.
The Pediatric resident on duty requested for a chest x-ray AP lateral which showed subgottic
narrowing or “steeple sign”
Guide Questions:
1.
2.
3.
4.
5.
6.
What are the essential aspects of the history and PE?
What are the differential diagnoses for ARI?
What is the most likely diagnosis?
What is the etiologic agent?
What are the useful diagnostic tools for the evaluation and diagnosis of PCAP?
What is the management for the different ARI?
PEDIATRIC CASE NUMBER 2-5
HISTORY: A 6-year-old male was brought to the ER because of difficulty of breathing. His
mother revealed that 2 days prior, he began to have cough with no other associated signs or
symptoms. She bought him carbocistein syrup and gave him 5 ml 3 times a day. On the
morning prior to consult, his cough was noted to worsen and was described as “barking cough”
with episodes of low to moderated grade fever. Three hours PTC, patient was noted to have
inspiratory stridor with difficulty of breathing, hence, was brought to the ER for consult.
PHYSICAL EXAMINATION:
GENERAL: Awake and in moderate respiratory distress. Weighs 19.5 kg.
VITAL SIGNS: HR 120, RR 45, and temperature 39°C.
HEENT: Anicteric sclerae, pinkish palpebral conjunctivae. No nasal discharge with positive
alar flaring.
Trachea in midline, positive cervical lymphadenopathy, negative
tonsillopharyngeal congestion, and with copious purulent sputum.
LUNGS: Symmetrical chest expansion with suprasternal retractions, positive ronchi and
occasional wheeze.
HEART: Regular cardiac rhythm, no murmur.
ABDOMEN: Flat, soft, non-tender, with normoactive bowel sounds.
EXTREMITIES: Grossly normal, full equal pulses, no cyanosis.
X-RAY: Chest x-ray showed subglottic narrowing with irregular appearing tracheal mucosa.
Guide Questions:
1.
2.
3.
4.
5.
6.
What are the essential aspects of the history and PE?
What are the differential diagnoses for ARI?
What is the most likely diagnosis?
What is the etiologic agent?
What are the useful diagnostic tools for the evaluation and diagnosis of PCAP?
What is the management for the different ARI?
PEDIATRIC CASE NUMBER 2-6
HISTORY OF PRESENT ILLNESS: A 6-year-old female was brought to the clinic due to
difficulty of breathing. Patient is a known asthmatic since 3 years of age. Three days prior to
consultation, she was noted to have watery nasal discharge with nonproductive cough. Few
hours prior to consultation, she experienced chest tightness and shortness of breath, hence, was
brought to the ER.
PAST MEDICAL HISTORY: Mother claimed that the patient had been having recurrent
episodes of cough with difficulty of breathing about 2 to 4 months a year since she was 2 years
old, usually occurring a few days after onset of colds. Last attack occurred 3 months ago. Her
mother also revealed that she would have bothersome cough when she would play tag with the
other children in the late afternoon but not more than 1 to 2 times a month. Occasionally, she
would get up in the middle of the night coughing, but would be relieved after just sitting up in
bed for a while, occurring not more than once a month. She was also hospitalized last year
because of difficulty of breathing.
BIRTH AND NEONATAL HISTORY: Delivered via spontaneous vaginal delivery to a 26year-old G2P1 mother with no complications. Exclusively breastfed for 1 year.
FAMILY HISTORY: (+) Bronchial asthma, maternal grandfather
PHYSICAL EXAMINATION:
GENERAL: Awake, ambulatory, talks in sentences.
VITAL SIGNS: Height 115 cm , weight 23 kg. HR 96, RR 40 per minute, temperature 38°C.
SaO2 is 96 %
Peak flow meter readings: (ideal = (115 – 100) x 5 + 170 (female) = 245
Pre-B2 agonist
Post-B2 agonist
190
205
195
215
200
225
200
(81.6 %)
230 (93.87 %) var 12.27 %
195
210
HEENT: Anicteric sclerae, pinkish palpebral conjunctivae. No ear discharge, positive watery
nasal discharge without alar flaring. Negative tonsillopharyngeal congestion, trachea in
midline.
LUNGS: Equal chest expansion, (-) retractions, (+) wheeze both lung fields. Regular cardiac
rhythm, no murmur.
ABDOMEN: Flat, NABS, non-tender.
EXTREMITIES: Essentially normal, full equal pulses.
Guide Questions:
1. What are the essential aspects of the history and PE?
2. What is the most likely diagnosis?
3. What are the differential diagnoses?
4. What is the definition of asthma?
5. What is the pathophysiology of asthma?
6. What are the useful diagnostic tools for the evaluation and diagnosis of asthma?
7. How is asthma and its exacerbation classified?
8. What are the goals of therapy, outcome, and general management of asthma?
9. How do you manage (immediate and advanced care) asthma exacerbations?
10. How do you manage on the long term children with asthma?
11. What are the various pharmacologic therapies used in the management of asthma?
12. What are the predisposing factors and triggers of asthma?
13. How do you control and prevent asthma?
PEDIATRIC CASE NUMBER 2-7
HISTORY OF PRESENT ILLNESS: A 5-year-old male was brought in because of cough and
difficulty of breathing. History revealed that a month prior to consultation, he had been having
attacks of cough and breathlessness occurring 3 times a week and would be awakened at night
time with the same symptoms occurring not more than 3 times a month. Symptoms were
temporarily relieved by taking salbutamol syrup 2mg/5ml, 5 ml 3x a day.
PAST MEDICAL HISTORY: His mother recalled that the patient began having recurrent
cough with difficulty of breathing 3 years ago, usually triggered by colds and exposure to dust
and strong odors. Last year he had similar symptoms and was admitted for 3 days. Home
medication recalled was budesonide MDI 200 mcg/puff, 1 puff 2x a day. However, this was
taken with good compliance for only 3 months because the medication was claimed to be
expensive and the patient was already symptom free.
BIRTH AND FEEDING HISTORY: Delivered live full term to a 28-year-old G1P0 mother via
spontaneous vaginal delivery. Breastfed for 2 months then shifted to NAN 1 – 1:1 dilution up to
6 months then NAN 2 – 1:1 dilution up to 1 year old.
FAMILY HISTORY: (+) Bronchial Asthma – Father
ENVIRONMENTAL HISTORY: No smokers in the family. To save on LPG they sometimes
use firewood for cooking.
PHYSICAL EXAMINATION:
GENERAL: Awake, prefers sitting, talks in phrases, irritable.
VITAL SIGNS: Height 108 cm, weight 17.5 kg. HR 116, RR 42 per minute, temperature 38.3°C.
SaO2 is 92 %.
Peak expiratory flow rates: ideal = (108 – 100) x 5 + 175 = 215
Pre-B2 agonist
165 (76.74 %)
160
160
155
160
HEENT: Anicteric sclerae, pinkish palpebral conjunctivae. No ear or nasal discharge, positive
alar flaring, (-) tonsillopharyngeal congestion, tachea in midline.
LUNGS: Equal chest expansion, (+) suprasternal and intercostals retractions, (+) wheezing over
both lung fields.
HEART: Regular cardiac rhythm, no murmur.
ABDOMEN: Flat, NABS, soft.
EXTREMITIES: Grossly normal, full pulses.
Guide Questions:
1. What are the essential aspects of the history and PE?
2. What is the most likely diagnosis?
3. What are the differential diagnoses?
4. What is the definition of asthma?
5. What is the pathophysiology of asthma?
6. What are the useful diagnostic tools for the evaluation and diagnosis of asthma?
7. How is asthma and its exacerbation classified?
8. What are the goals of therapy, outcome, and general management of asthma?
9. How do you manage (immediate and advanced care) asthma exacerbations?
10. How do you manage on the long term children with asthma?
11. What are the various pharmacologic therapies used in the management of asthma?
12. What are the predisposing factors and triggers of asthma?
13. How do you control and prevent asthma?
PEDIATRIC CASE NUMBER 2-8
HISTORY OF PRESENT ILLNESS: A 7-year-old female had been having cough with chest
tightness and breathlessness on a daily basis and is awakened 2 or more times a week during
nighttime for the last 2 weeks. However, daily activities were not affected. She was given
Terbutaline syrup 1.5 mg/5 ml, 5 ml 3 times a day and nebulization with salbutamol p.r.n. Few
hours prior to consult, she was noted to be tachypneic and was given nebulization with 1
nebule salbutamol but with no relief, hence, was brought in for consultation.
PAST MEDICAL HISTORY: At 2 years of age, patient started having recurrent episodes of
rhinorrhea, nasal obstruction and sneezing lasting for less than a month but with no
troublesome symptoms. Condition was relieved by intake of oral antihistamines and
decongestants. At 3 years of age, she was noted to have recurrent episodes of cough with chest
tightness and breathlessness. She had 2 hospitalizations 2 years ago and the most recent one at
12 months old. On discharge from her last admission, she was given Symbicort turbohaler, 2
puffs 2x a day. Six months ago, Symbicort was tapered and discontinued. Patient has been
asymptomatic since then.
BIRTH AND FEEDING HISTORY: Delivered live full term to a 30-year-old G3 P2 mother via
spontaneous vaginal delivery. Bottle-fed since birth with NAN HA1 1:1 dilution. After 2
months this was shifted to Isomil at 1:2 dilution.
FAMILY HISTORY: (+) bronchial asthma – mother
ENVIRONMENTAL HISTORY: Three weeks ago, construction was started to add a room to
their bungalow. Father is a smoker but claims not to smoke inside the house.
PHYSICAL EXAMINATION:
GENERAL: Awake, sitting up hunched forward, talks in words, irritable.
VITAL SIGNS: Height 115 cm, weight 21 kg. HR 132, RR 40 per min , temperature 38.1°C.
SaO2 = 88 %.
Peak expiratory flow rate: ideal = (115 – 100) x 5 + 170 = 245
140 (57.14 %)
140
140
135
135
HEENT: Anicteric sclerae, pinkish palpebral conjunctivae. No ear or nasal discharge, (+) alar
flaring, (-) tonsillopharyngeal congestion, trachea in midline.
LUNGS: Equal chest expansion, (+) suprasternal and intercostals retractions, (+) wheezing over
both lung fields audible without stethoscope.
HEART: Regular cardiac rhythm, no murmur.
ABDOMEN: Flat, NABS, soft.
EXTREMITIES: Grossly normal, (+) cyanotic nailbeds, (-) clubbing.
Guide Questions:
1. What are the essential aspects of the history and PE?
2. What is the most likely diagnosis?
3. What are the differential diagnoses?
4. What is the definition of asthma?
5. What is the pathophysiology of asthma?
6. What are the useful diagnostic tools for the evaluation and diagnosis of asthma?
7. How is asthma and its exacerbation classified?
8. What are the goals of therapy, outcome, and general management of asthma?
9. How do you manage (immediate and advanced care) asthma exacerbations?
10. How do you manage on the long term children with asthma?
11. What are the various pharmacologic therapies used in the management of asthma?
12. What are the predisposing factors and triggers of asthma?
13. How do you control and prevent asthma?
PEDIATRIC CASE NUMBER 2-9
HISTORY OF PRESENT ILLNESS: A 9-year-old male was brought to the emergency room
due to difficulty of breathing. Three months prior to consult, inhaled corticosteroids in the form
of Budesonide had been tapered off and discontinued. One month prior to admission, patient
was noted to have daily symptoms of chest tightness and breathlessness with night time
symptoms occurring 3x a week. He was being given salbutamol syrup 2mg/5ml, 5 ml 3x a day
with temporary relief. Two weeks PTA, he stopped going to school because he was easily
fatigued and preferred to stay in bed most of the time. Nebulization with salbutamol 1 nebule
every 4 – 6 hours. Two hours PTA, he was noted to have difficulty of breathing. He was
nebulized with salbutamol every 30 min x3 doses. Few minutes PTA, he was noted to be
gasping, hence, was rushed to the ER
PAST MEDICAL HISTORY:
BRONCHIAL ASTHMA: At 3 years old, the patient started having episodes of cough with
chest tightness and breathlessness. He had 2 previous admissions at 5 years old, 7 years old,
and 6 months ago because of difficulty of breathing. In the last admission he was prescribed
Seretide MDI (250 mcg fluticasone + 25 mcg salmeterol) 1 puff twice a day for 3 months then
shifted to Budesonide 200mcg/puff, 2 puffs 2x/day for 3 months more.
ATOPIC DERMATITIS: At 3 months of age he started to have erythematous, weepy patches on
the cheeks, with subsequent extension to the remainder of the face, neck, wrists, hands,
abdomen, and extensor aspects of the extremities. Condition was aggravated by the patient’s
efforts to rub face on bedclothes and sides of crib and later on by incessant scratching.
Condition was improved by application of emollients, steroid creams, and oral antihistamines.
This condition was noted to become less prominent after the age of 5 years. However,
occasionally there would be mild eczema that persists in the antecubital and popliteal fossae, on
the wrists, behind the ears, and on the face and neck.
ALLERGIC RHINITIS: At 2 years of age, he was noted to have began having recurrent episodes
of rhinorrhea, nasal obstruction and sneezing for less than a month but with no troublesome
symptoms. Condition was relieved by oral antihistamines and decongestants.
BIRTH AND FEEDING HISTORY: Delivered live full term to a 32-year-old G3P2 mother via
spontaneous vaginal delivery with no complications. Bottle fed since birth with S-26 1:2
dilution; however, this was discontinued after 2 weeks when patient developed facial rash. He
was shifted to Alfare for 3 months then shifted to NAN-HA 1 for the next 6 months then NANHA 2 until 12 months.
FAMILY HISTORY: (+) Bronchial Asthma –Father
(+) Allergic rhinitis – Mother
(+) Bronchial Asthma – older sibling
ENVIRONMENTAL HISTORY: Father is a smoker
PHYSICAL EXAMINATION:
GENERAL: Patient came in drowsy, sitting hunched forward supported by mother, and unable
in speak.
VITAL SIGNS : Height 125, weight 24 kg. HR 68 per minute, RR 12 per minute, temperature
36.4°C.
SaO2 = 76 %. Peak expiratory flow rate: ideal (125 – 100) x 5 + 175 = 300
HEENT: Anicteric sclerae, pale palpebral conjunctivae. (-) ear discharge (-) nasal discharge, (+)
alar flaring, (-) tonsillopharyngeal congestion, tachea in midline.
LUNGS: Equal chest expansion, (+) suprasternal and intercostals retractions, (+) chest
indrawing with thoraco-abdominal movement, poor air entry (-) wheeze.
HEART: Regular cardiac rhythm, no murmur.
ABDOMEN: Flat, NABS, soft.
EXTREMITIES: Grossly normal, (+) cyanotic nailbeds, (-) clubbing, thready pulses.
Guide Questions:
1. What are the essential aspects of the history and PE?
2. What is the most likely diagnosis?
3. What are the differential diagnoses?
4. What is the definition of asthma?
5. What is the pathophysiology of asthma?
6. What are the useful diagnostic tools for the evaluation and diagnosis of asthma?
7. How is asthma and its exacerbation classified?
8. What are the goals of therapy, outcome, and general management of asthma?
9. How do you manage (immediate and advanced care) asthma exacerbations?
10. How do you manage on the long term children with asthma?
11. What are the various pharmacologic therapies used in the management of asthma?
12. What are the predisposing factors and triggers of asthma?
13. How do you control and prevent asthma?
PEDIATRIC CASE NUMBER 2-10
GENERAL DATA: Patient is an 8-year-old male from Pasay City who sought consult last
November 10, 2005.
CHIEF COMPLAINT: Cough
HISTORY OF PRESENT ILLNESS: Three weeks PTC, the patient had low to moderate grade
undocumented fever associated with cough, productive of yellowish phlegm. He was positive
for cold and nasal congestion. Upon consult at the local health center, the patient was
diagnosed with an upper respiratory tract infection. The patient was prescribed with
amoxicillin 250 mg/5ml, 1 teaspoon 3x a day for 7 days, taken with good compliance.
However, there was no minimal improvement. Patient was lost to follow-up. Persistence of
symptoms prompted consult at PGH.
PAST MEDICAL HISTORY: At 5 years of age, patient was admitted at a local hospital for 5
days. Patient was diagnosed with bronchopneumonia. Patient was treated with unrecalled
antibiotics.
BIRTH MATERNAL HISTORY: Patient was born to a 23-year-old G2P1 (1001) mother via
SVD at home care of a traditional birth attendant. Mother had no regular prenatal check-ups
and allegedly had no maternal illness during gestation.
IMMUNIZATION HISTORY: None.
NUTRITIONAL HISTORY: The patient was breastfed until 2 years of age. Solid food was
introduced at 4 months of age. At present, patient’s usual meal consists of rice, instant noodles
and fish. Patient is fond of eating fish balls.
DEVELOPMENTAL HISTORY: At par with age. At present, the patient is a Grade 1 student
at Pasay public school with average scholastic standing.
FAMILY HISTORY: Negative DM and hypertension. Father and older sister has asthma and
maternal grandfather had TB and was treated for 6 months with 3 unrecalled antibiotics.
PERSONAL/SOCIAL HISTORY: The patient is the 2nd child in a brood of 3. Mother is a 28year-old laundrywoman and father is a 35-year old carpenter. Patient’s older sister is 9 years old
and patient’s younger brother is 3 years old. Patient’s family lives in a 1-room house in the slum
areas of Pasay. Maternal grandfather does not live with the patient’s family but resides in the
same vicinity.
REVIEW OF SYSTEMS:
(+) weight loss – approximately 25% in the past year
(+) poor appetite
(-) difficulty of breathing
(-) ear discharge
(-) cyanosis
(-) easy fatigability
(-) diarrhea
(-) abdominal pain
(-) joint pains
(-) lethargy
(-) seizures
(-) constipation
PHYSICAL EXAMINATION:
GENERAL: Patient is awake and active, not in cardiorespiratory distress. Weight is 22 kg and
height is 125 cm.
VITAL SIGNS: BP 90/60, HR 84, RR 20, Temperature 37.5°C.
HEENT: Pink conjunctivae, anicteric sclerae, positive large, matted posterior cervical
lyphadenopathy, non-tender, non-erythematous.
CHEST/LUNGS: Adynamic precordium, distinct heart sounds, regular rate and rhythm, no
murmurs. Equal chest expansion, clear breath sounds, no rales, no wheezes.
ABDOMEN: Abdomen is flat, soft, and negative for tendernaess, heptomegaly, and
organomegaly. Normoactive bowel sounds. Negative ascites.
EXTREMITIES: Pink nailbeds, full pulses, no edema or cyanosis.
GENITALIA: Normal external genitalia.
Guide Questions:
1. What are the possible differential diagnoses in a patient with chronic cough?
2. What are the diagnostic approaches to a patient with chronic cough?
3. How is a diagnosis of TB in children made? What are the diagnostic work-ups which
you will request for the patient? Pathophysiology of TB?
4. How will you treat the patient if proven to have pulmonary tuberculosis? Preventive
measures to control TB?
PEDIATRIC CASE NUMBER 2-11
HISTORY: Karel is 8 years old. She has been having cough for 3 weeks now. On the 1 st week
of her illness, there were no accompanying signs and symptoms. She was active, afebrile, had
good appetite, and remained playful. She was brought to her pediatrician at that time. Her
physical examination findings were normal if not for the occasional cough. No medication was
given to her. Her mother and she were advised that it was probably a self-limiting illness and
she was instructed bed rest and increased oral fluid intake. They did as told, however, her
cough persisted.
On the second week of her illness, she developed fever. She was also observed to have poor
appetite although she remained playful. Her mother gave her an antipyretic which afforded
temporary relief of fever. However, her mother observed that she had multiple lymph nodes at
the cervical area which bothered her, thus she was brought back to her pediatrician for another
consult.
PHYSICAL EXAMINATION: At the time of examination, her physician noticed that she had
fever documented at 38.5 C. She had multiple palpable lymph nodes at the cervical area. On
oropharyngeal examination, She had non-hyperemic tonsils and no exudates were noted. Her
physician inquired for any exposure to an adult with active tuberculosis, which her mother
denied. She then did a tuberculin test on Karel and they were advised to come back after 3 days
for the reading.
On follow-up consult, PPD was noted to have a 15 mm induration. At this time, Karel was on
her 3rd week of illness. She still had on and off fever, and she remained to have poor appetite.
Her physician noted that she has been losing weight.
Guide Questions::
1. What are the possible differential diagnoses in a patient with chronic cough?
2. What are the diagnostic approaches to a patient with chronic cough?
3. How is a diagnosis of TB in children made? What are the diagnostic work-ups which
you will request for Karel? Pathophysiology of TB?
4. How will you treat Karel if proven to have pulmonary tuberculosis? Preventive measures
to control TB?
PEDIATRIC CASE NUMBER 2-12
HISTORY: Alvin is a 7-year-old male with recurrent cough since 2 years ago. His cough
initially occurred episodically, around 3 to 4 times a year with worsening of symptoms
particularly during the rainy season and during cold weather. His mother noted that he would
have cough, shortness of breath accompanied by wheezing after playing with friends, and
during times of infection.
Six months ago, his cough started to become more frequent, this time accompanied by colds and
sneezing usually occurring early in the morning on waking up. His cough would get worse
during nighttime, leading to difficulty in sleeping. He would occasionally complain of
headaches. He has incurred a lot of absences from school because of this.
Persistence of Alvin’s symptoms prompted the mother to bring him to the OPD.
Guide Questions:
1. What OTHER important aspects in the history are important to ask?
2. What parts of the physical examination would be pertinent in this case?
Further history taking reveals the following:
He has constant throat clearing throughout the day with clear watery nasal discharge and
notices nasal obstruction occurring more often in the evenings. He was previously given
antibiotics (unrecalled), anthistamines, mucolytics and 2 agonists either singly or in
combination. These afforded only temporary relief, and sometimes the antihistamines have
made him drowsy, resulting in sleepiness during school hours.
IMMUNIZATION HISTORY: He completed his primary series of vaccinations at the local
health center.
FAMILY HISTORY: There is a positive maternal history of asthma. A grandmother living
with them was diagnosed as having “weak lungs” in the past but was treated more than 10
years ago and is not on any medications. Alvin’s father is a smoker.
PERSONAL/SOCIAL HISTORY: They live in a rented apartment, occupying only one room,
together with the father’s family. They have a pet dog living inside the house and the house
stands along a busy street.
PHYSICAL EXAMINATION:
VITAL SIGNS: Actual Body Weight = 19 kg
Actual height= 115cm
Heart Rate = 84 beats/minute
Respiratory rate = 28 breaths/minute
o
Temperature = 37 C
HEENT: (+) infraorbital darkening, (+) nasal crease, with pale, boggy mucosa with clear
rhinorrhea, cobblestone appearance of adenoidal tissue, no cervical lymphadenopathy.
CHEST/LUNGS: (+) very faint bilateral expiratory wheezes, no rales.
The rest of the examination is within normal limits.
Guide Questions:
1. Given the additional information, what is your diagnosis and differential diagnosis, if
any? Give the basis for each.
2. Given the above data, what initial diagnostic test(s) would you request for?
Chest x-ray showed normal results. PEFR monitoring showed the highest reading at 230L/min,
and lowest reading at 165L/min. Paranasal sinus series showed no opacification. Allergy skin
prick test revealed (+) results to house dustmite, dog dander and mixed molds.
Guide Questions:
1. What is your present working impression? (Basis for impression)
2. What will your approach to managing this patient be?
PEDIATRIC CASE NUMBER 2-13
HISTORY: A 15-year-old girl presented with prolonged coughing and wheezing attack which
had suddenly occurred a day before their class field trip to the National Museum. She was
unable to sleep during the night before because of this. Her mother noted a runny nose and
occasional eye itchiness a few days before.
Prior to this, she would have coughing episodes occurring 2 to 3 times a month mostly
accompanied by bouts of sneezing and at times, nasal stuffiness. About 3 months ago, she was
rushed to the ER in the middle of the night because of continuous coughing and difficulty
breathing.
Due to the persistence of symptoms she was brought to your clinic.
Guide Questions:
1. What OTHER important aspects in the history are important to ask?
2. What parts of the physical examination would be pertinent in this case?
Further history taking reveals the following:
During most days, she would have a runny nose, with clear watery discharge and would
occasionally wake up sneezing and scratching her nose. Consult with other pediatricians were
made and she was given various oral decongestant-antihistamine preparations which gave
temporary relief, but she noted that the symptoms would recur after a few days. She was also
prescribed oral 2 agonists, which relieved her coughing but she stopped taking them because it
gave her palpitations.
She had experienced episodes of “wheezy bronchitis” as a child and eczema as an infant. She is
a non-smoker. Her father suffered from hay fever but there was no family history of asthma or
tuberculosis. Her older brother is an occasional smoker.
She completed her primary series of vaccinations care of their pediatrician.
They live in an exclusive village in the South, and she has her own room, with air-conditioning
and a carpet. They have a pet dog living inside the house and the house is far from the main
road.
PHYSICAL EXAMINATION:
VITAL SIGNS: Actual Body Weight = 34 kg
Actual height= 150cm
Heart Rate = 92 beats/minute
Respiratory rate = 28 breaths/minute
o
Temperature = 37 C
HEENT: (+) infraorbital darkening, (+) nasal crease, with pale, boggy mucosa with clear
rhinorrhea, cobblestone appearance of adenoidal tissue, no cervical lymphadenopathy.
CHEST/LUNGS: (+) bilateral expiratory wheezes, no rales.
The rest of the examination is within normal limits.
Guide Questions:
1. Given the additional information, what is your diagnosis and differential diagnosis, if
any? Give the basis for each.
2. Given the above data, what initial diagnostic test(s) would you request for?
Chest x-ray showed normal results. PEFR monitoring showed the highest reading at 320L/min,
and lowest reading at 250L/min. Paranasal sinus series showed no opacification. Allergy skin
prick test revealed (+) results to house dustmite, dog dander and cockroach.
Guide Questions:
1. What is your present working impression?
2. What will your approach to managing this patient be?
PEDIATRIC CASE NUMBER 2-14
HISTORY: Joanne is a 10-year-old female who presents to your clinic with a history of asthma
and year-round symptoms of rhinitis. She has wheezing and coughing at least 2 times per week
but less than once a day. She occasionally has nighttime coughing. She uses a short-acting 2
agonist several times per week that stops her wheezing but it returns quickly. Her symptoms of
asthma started when she was 4 years of age. These symptoms include clear rhinorrhea,
sneezing and congestion. These symptoms occur throughout the year. She regularly takes
over-the-counter antihistamines which help relieve her symptoms as long as she takes it daily.
Her nasal symptoms were worse in the last year and are associated with poor school
performance.
Guide Questions:
1. What OTHER important aspects in the history are important to ask?
2. What parts of the physical examination would be pertinent in this case?
Further history taking reveals the following:
The patient’s mother has seasonal nasal symptoms and the patient’s father smokes cigarettes.
They have a dog that Joanne adores, who sleeps with her. The patient had atopic dermatitis
and food allergy during the 1st 2 years of life.
Her aunt, who lives with their family, was allegedly treated with anti-TB medications for 6
months a few years ago.
PHYSICAL EXAMINATION:
VITAL SIGNS: Actual Body Weight = 33 kg
Actual height= 130cm
Heart Rate = 84 beats/minute
Respiratory rate = 18 breaths/minute
o
Temperature = 37 C
HEENT: (+) infraorbital darkening, (+) nasal crease, with pale, boggy mucosa, bilaterally with
clear rhinorrhea; (-) polyps noted, cobblestone appearance of adenoidal tissue, (+) cervical
lymphadenopathy.
CHEST/LUNGS: (+) faint bilateral expiratory wheezes, no rales.
The rest of the examination is within normal limits.
Guide Questions:
1. Given the additional information, what is your diagnosis and differential diagnosis, if any?
Give the basis for each.
2. Given the above data, what initial diagnostic test(s) would you request for?
Chest x-ray showed normal results. PEFR monitoring showed the highest reading at 230L/min,
and lowest reading at 180L/min. Paranasal sinus series showed no opacification. Allergy skin
prick test revealed (+) results to house dustmite, dog dander and cockroach.
Guide Questions:
1. What is your present working impression?
2. How would you classify the severity of this patient’s asthma and rhinitis?
3. What will your approach to managing this patient be?
PEDIATRIC CASE NUMBER 2-15
HISTORY: Anna is a 17-year-old female with complaints of “sinusitis” since 2 years ago. Her
acute problem began 1 week earlier with symptoms of a “cold”, including sneezing, rhinorrhea,
sore throat, malaise and low-grade fever. After several days, she began experiencing facial pain
and pressure located more over the right cheek bone. Her nasal discharge was initially watery,
becoming thicker, yellow and then green. She awakens at night, coughing and experiencing a
sore throat upon waking up in the morning. She feels generally fatigued.
Persistence of Anna’s symptoms prompted her mother to bring her to your clinic.
Guide Questions:
1. What OTHER important aspects in the history are important to ask?
2. What parts of the physical examination would be pertinent in this case?
Further history taking reveals the following:
On further questioning, Anna has been having 2 to 3 episodes each year for the past 2 years and
this is the 3rd such episode this school year. Although she considers herself to be otherwise
healthy, she admits she experiences nasal stuffiness, a need to blow her nose several times in a
day after showering and a frequent sensation of drainage of mucus from her nose into her
throat. She clears her throat often as well. There seems to be no seasonal variation in her
symptoms although the attacks occur primarily during the cold months. Her mother and
brother have recurrent sinus infections and her brother also has hay fever and mild persistent
asthma. Anna owns a cat which she acquired 5 years ago but she denies that her symptoms
have increased on close contact with the cat, but they seemed to have gotten worse.
PHYSICAL EXAMINATION:
VITAL SIGNS: Actual Body Weight = 40 kg
Actual height= 150cm
Heart Rate = 84 beats/minute
Respiratory rate = 18 breaths/minute
Temperature = 37oC
HEENT: (+) infraorbital darkening, no aural discharge, intact tympanic membrane, (+) nasal
crease, with inflamed, swollen mucosa with thick, light green mucus in the nasal cavity, (+)
similar discolored mucus on the posterior pharyngeal wall with cobblestoning and erythema,
no cervical lymphadenopathy.
CHEST/LUNGS: Clear breath sounds, no wheezes, no rales.
The rest of the examination is within normal limits.
Guide Questions:
1. Given the additional information, what is your diagnosis and differential diagnosis, if
any? Give the basis for each.
2. Given the above data, what initial diagnostic test(s) would you request for?
Chest x-ray showed normal results. PEFR monitoring showed the highest reading at 380L/min,
and lowest reading at 370L/min. Paranasal sinus series showed opacification in the R maxillary
area. Allergy skin prick test revealed (+) results to house dustmite, cat dander and cockroach.
Guide Questions:
1. What is your present working impression? (Basis for impression)
2. What will your approach to managing this patient be?
DEVELOPMENTAL
DELAY
PEDIATRIC CASE NUMBER 3-1
HISTORY: Alex is a 3-year-old boy from Bulacan who was brought by his mother because of
difficulty to walk independently. He was delivered 7 months premature to a 26-year-old G2P1
mother at a local hospital after an uncomplicated pregnancy. He weighed 900 gm at birth and
was placed in an incubator and required oxygen because of episodes of cyanosis and apnea. He
received blood transfusion because of pallor but no jaundice or seizures were noted. Patient
was discharged after 3 months with a weight of 1.2 kg. While at home, the mother noted that he
seemed to be a floppy baby. He could not turn over nor hold his neck up even at 5 to 6 months.
At around 8 months of age, he assumed spastic flexion of both upper extremities with fisting of
hands and extension of both lower extremities. He was unable to hold his bottle or reach for or
grasp objects. He only cried then and did not yet utter any word. No consult was made at that
time. When patient turned 14 months old, he was brought to a local hospital and he was
advised physical rehabilitation for his spasticity. Physical therapy was done twice a week for
almost 1 year. Despite therapy, he was only able to lift his head at 18 months and sit alone at 2
years old. He had difficulty reaching out for toys because of spasticity of his arms. He was also
described to be drooling his saliva and choked on solid foods. He only tolerated soft or mashed
food and milk given in small quantities. Therapy was done at home when he was 2½ years and
he was noted to say his first word “mama” just before he turned 3 and now combined syllables
like “bada”, “dede”, and “tata”. When supported, he could stand on tiptoe position but could
not yet stand alone. His legs and arms were still spastic which prompted mother to seek
consult.
PAST MEDICAL HISTORY: He was diagnosed with primary complex at 6 months of age and
was given anti-Kochs medications for 6 months. He received primary immunizations at the
health center without booster doses.
PERSONAL/SOCIAL HISTORY: Patient is the 2nd of 4 siblings. His father is 29 years old and
works as a fishpond caretaker while his mother is a 31-year-old housewife. Both are high school
graduates. Other siblings are healthy.
PHYSICAL/NEUROLOGICAL EXAMINATION: Physical and neurological examination
reveals the following anthropometric measurements: Weight of 11 kg, height of 93 cm, head
circumference 46.5 cm. He was awake and active, smiled often when spoken to and would
occasionally babble. He was observed to have drooling of his saliva. He had an essentially
normal systemic examination except that his arms and legs were thin and were spastic, resistant
to flexion. His fingers were also extended and he had difficulty moving them. His legs were
extended and he stood up with support on tiptoes, walked with a scissoring gait while he was
supported under his arms as he could not yet walk alone. He cried in response to painful
stimuli applied to his fingers and toes. The reflexes could not be elicited due to persistent
spasticity of the extremities. There was bilateral babinski but no ankle clonus was elicited.
Guide Questions:
1.
2.
3.
4.
5.
6.
7.
What is the presenting problem of the patient?
What are the evidences of developmental delay in the patient?
What are the normal ages to attain the above-mentioned milestones.
What is cerebral palsy?
What are the risk factors for cerebral palsy?
How is cerebral palsy classified? As to impairment and as to topography.
What are the associated problems in cerebral palsy? So what are the diagnostic measures
required to evaluate for associated impairments?
8. What are the principles or goals of intervention for children with cerebral palsy?
PEDIATRIC CASE NUMBER 3-2
HISTORY: Nonong is a 3-year- and 9-month-old male from Quezon City who was brought to a
pediatrician due to delayed development and frequent jerking episodes. He had a stormy
postnatal course. He was born to a 30-year-old mother, her 3rd pregnancy. She had hypertension
during her 9th month of pregnancy with no intake of medications. This complicated her
delivery, necessitating assistance via outlet forceps extraction. He had poor Apgar score (2  5
7) such that resuscitation was done with face mask ventilation and was eventually intubated.
He was admitted to the NICU and treated as a case of hypoxic ischemic encephalopathy and
neonatal pneumonia. He had seizures characterized as cycling movements of extremities and
was given phenobarbital. He was also given antibiotics for the pneumonia. He was gradually
weaned from the ventilator and was finally discharged after 2 months.
Upon discharge, his mother observed episodes of spasms occurring almost daily for a few
seconds. No medications were taken then. He was seen for follow up at 1½ months and
assessed to have epilepsy; phenobarbital was resumed. Cranial ultrasound performed at 2
months of age showed mild ventriculomegaly with cerebral atrophy. Electroencephalograph
done at the same time revealed presence of intermittent delta slowing of background indicative
of a focal cerebral dysfunction of nonspecific etiology.
The patient was then brought for regular follow up with a private physician until 6 months of
age then was lost to follow up. It was observed by the mother that he did not have visual
threat. He did not regard his mother or follow moving toys in front of him. It was also
becoming evident later on that he was not developing at par for his age. Spasticity of the lower
extremities was noted at 4 months of age; during this time he still had no head control and his
hands were still fisted. Although he could localize sounds, no other improvements were noted.
At the same time, he still had occasional episodes of tonic seizures. While he was growing,
spasticity on his upper and lower extremities was becoming more severe; no further progress in
his development was noted. Medical consults were discontinued due to financial constraints.
Except for the seizures, there were no other serious medical conditions.
His mother is a single parent, a high school graduate and works as a beautician. He lives with
his mother and an older brother who is 8 years old and attending regular school.
During his most recent visit, the patient still had poor head control and could only turn his head
on one side. He could momentarily grasp objects, localize to sound but had no babbling and
only cried to indicate needs. He could smile spontaneously and expressed excitement.
PHYSICAL EXAMINATION: Pertinent PE findings were a weight of 8.3 kg, height of 85 cm,
head circumference of 45 cm. He had several dental caries and was drooling saliva. His chest
and cardiac findings were normal. His abdominal muscles were tight when examined. On
neuro exam, he was awake and active but not verbal. He would only cry or squeal in
excitement. All his extremities were spastic with contractures noted due to difficulty to move
his knee, ankle, elbow and wrist joints. He responded to pain by slight movement. DTRs were
+++ on the lower extremities with bilateral babinski but with no ankle clonus.
PEDIATRIC CASE NUMBER 3-3
HISTORY: Anne is a 4-year- and 9-month-old female child from Muntinlupa City who was
brought to her pediatrician because of speech delay. Her mother became worried when at 12
months of age, she did not say “mama” or “papa”. She was also noted to be unaffected when
approached by other unfamiliar adults. At 18 months of age, she babbled but had no words.
She also did not point or make any gestures when she wants something from her mother. She
did not point to her body parts when asked or requested to do, and when talked to by other
people, she only had brief eye contact. At 2 years of age, she was noted to be unusually
fascinated by electric fans, wheels and anything round and rotating. At this time, she was able
to say “mama” and “dada” but these words do not pertain to her parents. When she was
brought to the playground or parties to mingle with other children her age, she did not join
them in games or any activities; instead, she preferred playing alone with beads, sorting them
by colors and lining them up. She did not engage in interactive play with children.
At 3 years of age, the patient began to use 2-word phrases such as “Anne big!” and other
phrases that she heard from television commercials. Her mother also noticed that when Anne
was injured, she did not seek her mother’s comfort and showed high tolerance for pain. At 4
years, she did not respond to verbal requests or maintain eye contact. When she was given a
toy car, she would repeatedly spin its wheels and flap her hands.
She reportedly reached her motor milestones “on time”.
Mother had an apparently normal pregnancy and delivery. As an infant, Anne seemed quite
normal and healthy.
PHYSICAL EXAMINATION:
unremarkable.
Physical and neurological examinations were essentially
PEDIATRIC CASE NUMBER 3-4
HISTORY: Kiel is a 3-year- and 2-month-old boy from Las Pinas City, who was brought to the
outpatient clinic by his mother due to speech delay. Mother started to have concerns when at 9
months old, it was difficult for her to get his attention. He would not respond when his name
was called or when his siblings are busy playing around, he would just sit alone and play with
his toys. His parents thought he could not hear but he would respond to his favorite
commercials on television and cover his ear to rock music heard over the radio. During this
time, he was not yet babbling as well although he was already beginning to stand while holding
on to something. At 1 year old, his mother noticed that he started babbling; however, he often
would cry to indicate his needs and the mother could only guess what he was trying to
communicate because he did not point. He only pulled the hand of his mother to get things for
him at times. As he was growing older, they noticed Kiel speak several unintelligible words
which sounded like a foreign language. He was unable to follow commands given to him even
when demonstrated.
At 2 years of age, there was no improvement in his language skills. He was very active and
kept running around in circles and would always jump and laugh aloud; his mother thought he
had a very happy disposition. When he wanted something, he would pull the hand of his
mother to get it for him or he would get things by himself even if it required him to climb up
dangerous places. When hurt, he would cry but would not seek comfort from other people. He
would run around with his younger sibling but most of the time preferred to play by himself.
He loved spinning the wheel of his bicycle and arranged his toys in a linear fashion, and that if
his toys were disarranged, he would go into tantrums. This continued until 3 years of age;
hence, his mother decided to seek consult. During this time, he still did not have any
intelligible word and did not seem to understand commands asked because he would rarely
comply. He could not yet eat by himself and was not yet toilet trained.
BIRTH/MATERNAL HISTORY: The patient is a product of an uncomplicated full term
pregnancy of a 21-year-old primigravid. He was delivered in the hospital via spontaneous
vaginal delivery with a birth weight of 3 kg. His mother had UTI during the 4 th week of
gestation for which she took amoxicillin for 1 week. His neonatal and infancy period were
unremarkable with no history of hospitalization since.
SOCIAL/PERSONAL HISTORY: Patient is the eldest of 2 siblings. His father is 28 years old,
an electronic operator and the mother is 24 years old, a billing clerk. Younger sibling is a 1-year
old-boy who is observed to have better language skills than Kiel. His mother reports a firstdegree cousin(father’s side) who also still has poor social and language skills at the age of 5.
PHYSICAL EXAMINATION: Weight, height and head circumference were within the 50th
percentile. Systemic physical exam was unremarkable. There were no noted dysmorphisms or
cutaneous stigmata. Neurologic examination was likewise unremarkable.
Guide Questions:
1. What is the Presenting problem?
a. What are the normal language milestones in young children?
b. What are the evidences of speech delay in this patient?
2. What conditions are we considering given the case?
3. What is Autism?
4. What is the underlying cause or pathophysiology of the condition?
5. How is Autism diagnosed? What is the triad of impairments?
6. What are the goals of intervention?
FAILURE
TO THRIVE and
SHORT STATURE
PEDIATRIC CASE NUMBER 4-1
HISTORY: An 8-month-old female from Sta. Ana was brought to the outpatient department
for poor weight gain. History of present illness revealed that she was born term via
spontaneous vaginal delivery to a 30-year-old G3P2 (2002) mother at the local health center
assisted by a midwife, with a birth weight of 3.1 kg. There were no perinatal complications.
The mother had neither illness nor intake of medications other than multivitamins during her
pregnancy. Starting at 2 months of age, the patient was noted to have intermittency of
diaphoresis while feeding, and had frequent visits to the local health center for URTI (upper
respiratory tract infection); however, no episodes of cyanosis were noted. Five days prior to
consult, the patient was noted to have colds with clear nasal discharge. Three days prior to
consult, she developed fever and cough productive of whitish phlegm for which consult was
made at the local health center where she was given paracetamol and carbocisteine, and advised
consult to our institution for further work-up of poor weight gain. Patient has already received
the following immunizations: 1 BCG, 2 OPV, 2 DPT. She was breastfed for a month then given
Bonna. Family medical history was unremarkable.
PHYSICAL EXAMINATION:
GENERAL: Physical examination revealed that she is irritable, tachypneic, and diaphoretic.
VITAL SIGNS: Her HR is 170 per minute, RR 60 per minute, BP 80/40 mmHg, temperature of
37.8°C, and O2 saturation of 96%. She weighs 4 kg and has a length of 62 cm.
HEENT: She has pink conjunctivae, anicteric sclerae, (+) cervical lymphadenopathy, no
tonsillopharyngeal congestion, no neck vein engorgement.
CHEST/LUNGS: She has bibasal rales with wheezing. Chest findings showed a dynamic
precordium, (+) slight precordial bulge (+) Harrison’s groove, tachycardic, DHS, regular
rhythm, (+) grade 3/6 holosystolic murmur at the LLSB.
ABDOMEN: The abdomen is soft, with normoactive bowel sounds, and liver edge palpable at
3cm below the right costal margin.
EXTREMITIES: She has pink nail beds with full and equal pulses, no edema was noted.
Guide Questions:
1. Is there failure to thrive in this patient?
2. What are the features in the history and physical examination which suggest a possible
cardiac cause of failure to thrive?
3. What is the pathophysiology of Congestive heart failure (CHF)?
4. What is your working diagnosis? Differential diagnosis? Basis?
5. What initial laboratory tests would you request for and why? What are your expected
findings?
X-RAY: Chest X-ray shows cardiomegaly with left atrial & ventricular enlargement, increased
PVMs & infiltrates.
LABORATORY: CBC shows elevated WBC with PMN predominance.
electrolytes are normal.
CBG and serum
ELECTROCARDIOGRAM: ECG shows sinus tachycardia, left axis deviation & left ventricular
hypertrophy.
Guide Questions:
1.
2.
3.
4.
What is your working diagnosis now?
How do you confirm your diagnosis?
What would you do next?
What are the non-pharmacologic and pharmacologic management which you will give
this patient?
a. Identify and prioritize problem list.
b. How would you manage the identified problems in this patient? Give the rationale
and necessary precautions for your management.
PEDIATRIC CASE NUMBER 4-2
HISTORY: A 3-year-old girl was admitted because of chronic cough and failure to thrive.
Symptoms started about 2 months prior to consultation, when she started to cough. She was
brought to the local health center, and was given oral amoxicillin for 7 days. There was however
progression of the cough and her appetite became poor. There was low-grade fever, relieved by
the intake of paracetamol. Because of difficulty of breathing, she was brought to the Sick Child
Clinic.
PAST MEDICAL HISTORY: Past medical history revealed that she had frequent episodes of
cough and colds, relieved with intake of mucolytics. Her maternal grandmother is presently on
anti-Koch’s regimen. The patient eats regular table foods; however, the family only eats 2 meals
a day. At 6 months of age, she weighed 5 kg, 8 kg at 1 year, and 9 kg at 2 ½ years.
PHYSICAL EXAMINATION: At the clinic, she was noted to be in respiratory distress with
HR=120/min; RR=43/min; T = 38.30C. Her weight was 9 kg and her height was 80 cm. Her midupper arm circumference was 10 cm. She had pale conjunctivae with bilateral multiple cervical
lymphadenopathies. Chest findings revealed rhonchi and rales all over both lung fields. Her
heart sounds were distinct with a normal rate and regular rhythm. The abdomen was soft with
no masses or organomegaly. There was no pedal edema and the pulses were full.
Guide Questions:
1. Are there pieces of information in the history and physical examination that you find
lacking? Enumerate.
2. What is your clinical impression and the basis for it? Is the child failing to thrive? Include
complete anthropometric assessment of her nutritional status.
3. What are the predisposing factors to the development of the child’s current nutritional
state?
4. Outline the management approach to this patient to include both therapeutic and
preventive interventions.
PEDIATRIC CASE NUMBER 4-3
A 10-year-old male from Quezon was referred at the Endocrine OPD because of short stature
with developmental delay. Birth history revealed prolonged jaundice. During infancy, his
mother claims that he was a quiet baby because he seldom cries. He has been constipated since
2 months old. His mother thought that he was constipated because of his feeding difficulty due
to his large tongue. At 5 months old, he was noted to look different from the rest of the family
members. His 1ST tooth erupted at 1½ years old. His developmental milestones were that of a
4-year-old boy.
On physical examination, he was awake with CR of 70, RR 23, afebrile. He weighs 30kg and
measures 115 cm in height (height age of 6 yrs old). He has coarse and dry hair and puffy
eyelids with multiple dental caries. Heart and lung findings were essentially normal. He has
globular abdomen with umbilical and inguinal hernia.
Laboratory Findings are as follows:
FT4
4.2 pmol/L (NV 11-24)
TSH > 100 mIU/L ( NV 0.3-3.8)
Bone age : Greulich Pyle: standard male of 5 y/o
Guide Questions:
1. What other information would you like to elicit from the history? Why ?
2. What other physical findings would you look for? Why?
3. Plot your anthropometric indices and interpret the chart.
4. What is the most likely diagnosis?
5. What is the basis for your diagnosis?
6. What are the possible etiologies of your diagnosis?
7. What diagnostic tests will you request?
8. How will you manage this patient?
9. What is the prognosis of this patient?
10. Is it possible to diagnose the patient’s condition based on PE at birth?
PEDIATRIC CASE NUMBER 4-4
GENERAL DATA: A.G., an 8-year-old female from Punta, Calamba, Laguna was admitted at
UP-PGH due to short stature.
HISTORY OF PRESENT ILLNESS: A year and a half PTA, the patient had headache
accompanied by vomiting. There were no other associated symptoms. Six months PTA there
was progression in frequency of headache and vomiting. Five months PTA, consult was done
at PGH–OPD wherein a cranial CT scan was requested, the result of which was normal.
Diagnosis was migraine headache with which she was prescribed with Ibuprofen. Three
months PTA, the patient was noted to have pallor accompanied by weight loss. Consult was
done and she was given with unrecalled medications. Three weeks PTA, she was confined in a
local hospital in Laguna and underwent blood transfusion where she was discharged after
improvement of condition. However, due to short stature she was advised to seek consult.
PHYSICAL EXAMINATION:
VITAL SIGNS: BP 180/110, RR 56, HR 140s, T 36.7.
BW 14 kg weight for height 18.57 ~ 75%, moderately wasted
Ht 95 cm ideal height for age 116.8 cm ~ 81%, severely stunted
HEENT: Normocephalic, no periorbital edema. Pale conjunctivae, anicteric sclerae.
CHEST: Symmetrical chest expansion, (-) retractions, no rales/wheeze.
HEART: Adynamic precordium, tachycardic, (+) murmurs.
ABDOMEN: No abdominal masses.
EXTREMITIES: No pedal edema.
LABORATORY EXAMINATION:
CBC: Hgb 73
hct .21
plt 180
seg 0.66
lymphos 0.19
mono 0.10
eo 0.03
ELECTROLYTES: Na 141 mmol/L ( 140–148 mmol/L)
K 4.3 mmol/L ( 3.6–5.2 mmol/L)
phosphorus 1.62 mmol/L ( 0.81-1.58 mmol/L)
BUN 45 mmol/L ( 2.60–6.40 mmol/L)
crea 1199 mmol/L (53–115 umol/L )
alb 27 g/L ( 34–50 g/L)
ABG: pH 7.35
pCO2 18.9
pO2 71
HCO3 10.70
BE -11.8
sats 94.1%
URINALYSIS: yellow/hazy
Sp gr: 1.010
pH : 6.5
Sugar: negative
protein: 2+
RBC 0-3/hpf
cast: negative
WBC 1-3/hpf
crystals: negative
epithelial cells rare
bacteria: rare
KUB ULTRASOUND: Kidneys are small with marked increased in echogenicity and poor
corticomedullary differentiation.
The R kidney measures 5 cm x 3.7 cm x 3.7 cm with parenchymal thickness of 0.8 cm.
The L kidney measures 5.1 cm x 3.4 cm x 3.5 cm with parenchymal thickness of 0.9 cm.
The central echo complexes are intact. No lithiasis seen.
The urinary bladder is partially filled, precluding its adequate assessment.
DIARRHEA
PEDIATRIC CASE NUMBER 5-1
A 3-year-old girl presented with bloody diarrhea and fever for 5 days, associated with altered
sensorium a day prior to consultation.
Clinical findings revealed an obtunded child, hypotonia, and pallor. She was 90 cm long,
weighed 11.5 kg, and had a head circumference of 49 cm. She had very sunken eyeballs, dry lips
and oral mucosa. Skin retraction was very slow. Pulses were weak.
Laboratory examinations were as follows: hemoglobin 9.5 g/dL, WBC 6.8 × 109/L with 60%
polymorphs and 40% lymphocytes, platelet count 320 ×109/L. The serum creatinine was 1.4
mg/dL and urine albumin was trace. C-reactive protein was 6 mg/L. Buffy coat smear showed
gram-negative bacilli. Blood culture grew S. dysenteriae type 1, which was sensitive to
gentamicin, amikacin, norfloxacin, and nalidixic acid and was resistant to amoxycillin,
chloramphenicol, tetracycline, and cotrimoxazole.
She was subsequently treated with intravenous fluids using a combination of D5LR and D50.3
NaCl and parenteral ampicillin and gentamicin.
Guide Questions:
1. What is the likely etiologic agent causing diarrhea in this infant? Justify your answer by
citing the clinical features that are consistent with it.
2. What is the level of dehydration in this patient? Justify your answer.
3. What is the most appropriate fluid management for this child? Comment on the
management given.
4. Are there any other complications of diarrhea besides dehydration? If so, how would
you manage this?
5. Besides fluid management, what other therapy should be instituted?
6. What take home advice would you give to the mother?
PEDIATRIC CASE NUMBER 5-2
A 1-year-old, bottlefed infant was brought to the emergency room because of diarrhea and
vomiting. Symptoms developed 24 hours prior to consultation when he was noted to pass
watery stools for 8 times about ½ cup per episode. This was associated with low-grade fever of
380C relieved by intake of paracetamol. He was given ORS but he kept vomiting with each
intake.
On physical examination, he was irritable with sunken eyeballs and dry lips. He weighed 7 kg
and is 75 cm long. Skin pinch retracted slowly. His abdomen was slightly distended with
hyperactive bowel sounds. He was subsequently hydrated with intravenous fluids. Fecalysis
done revealed no significant findings while rotavirus stool antigen test was positive. He was
discharged after 2 days with occasional episodes of passage of soft stool.
Guide Questions:
1. What is the likely etiologic agent causing diarrhea in this infant? Justify your answer by
citing the clinical features that are consistent with it.
2. What is the level of dehydration in this patient? Justify your answer.
3. What is the most appropriate fluid management for this infant? Comment on the
management given.
4. Are there any other complications of diarrhea besides dehydration? If so, how would
you manage this?
5. What take home advice would you give to the mother?
PEDIATRIC CASE NUMBER 5-3
A 1-year-old boy was brought in for diarrhea of 1-month duration. His mother recalled that
about a month prior, he had a bout of gastroenteritis with fever and dehydration prompting
consult at the nearest polyclinic. She recalled being advised oral rehydration salts, zinc
supplement, lots of fluids and a low-fat diet. She gave him lots of apple juice everyday since
then. She however noticed that his stools were loose and watery on some days especially when
given his usual milk formula, and mushy on other days. For the past week, he has had 6 to 10
episodes of watery stools per day, notably explosive upon milk intake. She also noted some
weight loss.
In the ER, he was noted to be active, alert, and playful. He weighed 9.5 kg with a length of 80
cm. Capillary refill was <2 seconds. He had moist lips and oral mucosa, skin pinch goes back
very quickly, and pulses were full. He seems to be uninterested when offered a drink.
Guide Questions
1. What is your primary clinical impression?
2.
Justify your impression above.
3.
What advise will you give his mother?
4.
What are the other non-infectious causes of diarrhea in this age group?
MUSCULOSKELETAL
PAIN
PEDIATRIC CASE NUMBER 6-1
An 8-year-old male was brought to the outpatient department for knee pain and swelling.
History started 2 weeks ago when he had intermittent fever of 38.5°C to 40°C. Two days later,
there was erythematous, painful swelling of his right knee spontaneously resolved after 2 days.
One week prior to consult, he had body malaise, easy fatigability and elbow pains. Five days
prior to consult, he again had painful swelling of the left knee. Persistence of symptoms
prompted consultation to our institution.
PE on consult revealed: HR 120/min, RR 20/min, T 37.7 C, conscious, coherent, with antalgic
gait, pale, no neck vein engorgement, not in respiratory distress, clear breath sounds, adynamic
precordium, PMI/AB at 5th ICS LMCL, distinct heart sounds, tachycardic, regular rhythm, no
murmur, no hepatomegaly, full and equal pulses, (+) multiple 1 cm firm, non-tender nodules on
both olecranon, right knee with warmth, pain and swelling.
Guide Questions:
1. What additional information in the history would you ask for and why?
2. What specific physical findings would you look for and why?
3. What is your initial impression? Differential diagnoses? Bases?
4. What initial laboratory tests would you request and why? What are your expected
findings?
5. What is the etiopathogenesis of acute rheumatic fever?
CBC shows anemia and leukocytosis. Chest x-ray is normal. ECG shows sinus tachycardia,
normal axis with no chamber enlargement, prolonged PR interval. ASO, ESR and CRP are
elevated. Throat culture is negative.
Guide Questions:
1. What is your working diagnosis now?
2. Differentiate rheumatic fever from juvenile idiopathic arthritis.
3. What would you do next?
4. Identify and prioritize problem list.
5. How would you manage the identified problems in this patient? Give the rationale and
necessary precautions of your management.
PEDIATRIC CASE NUMBER 6-2
A 5-year-old female was brought to the outpatient department clinic because of a 10-month
history of knee and leg pains. The pain usually occurs at night and would sometimes awaken
the patient due to the intensity of the pain. The pain subsides spontaneously or after the mother
massages the affected leg. The pain occurred initially once every 2 months but increased in
frequency in the last month to once a week, thus prompting consult.
On PE, patient’s vital signs were as follows: HR=100 RR=24 T=36.8 *C. She was awake and alert.
Patient had pink conjunctivae and small bilateral cervical lymphadenopathy. Heart and lung
findings were essentially normal. Patient walks normally.
Guide Questions:
1.
2.
3.
4.
What additional information in the history would you ask for and why?
What specific physical findings would you look for and why?
What initial laboratory tests would you request for and why?
What is your initial impression? Differential diagnoses for musculoskeletal pain? Basis?
The child’s pain usually affects either the right or left leg at a time. It usually lasts for less than
half an hour and resolves spontaneously or after the mother massages the leg. Patient walks
normally upon waking up in the morning after the episode of the leg pain. Patient has no other
subjective complaints and is essentially well. The musculoskeletal PE was essentially normal.
CBC showed normal hemoglobin, white blood cell count and platelet count. ESR is normal. The
x-rays of the lower limb showed no significant osseous findings.
Guide Questions:
1. What is your working diagnosis now?
2. What will you do next?
3. How will you advise the parents?
HEADACHE
PEDIATRIC CASE NUMBER 7-1
GENERAL DATA: Patient is a 14-year-old Filipino male and a student from Manila.
CHIEF COMPLAINT: Headache.
HISTORY OF PRESENT ILLNESS: One month PTC, patient experienced on and off bifrontal
headache lasting for several minutes which was relieved by paracetamol intake. No consult
was done.
PAST MEDICAL HISTORY: Patient is negative for trauma. No previous hospitalization.
BIRTH MATERNAL HISTORY:
complications.
Born FTSVD to a 38-year-old G3P3, (-) fetomaternal
IMMUNIZATION HISTORY: Positive BCG, DPT3-OPV3, and HepB3
NUTRITIONAL HISTORY: Breastfed for 4 months then mixed feeding thereafter; presently
no food preferences.
DEVELOPMENTAL HISTORY: At par with age.
FAMILY HISTORY: Grandfather has had stroke and mother is positive for migraines.
PERSONAL/SOCIAL HISTORY: Patient is a 3rd year high school student who has no vices.
He is the eldest of 3 children to a 58-year-old housewife and a 54-year-old government
employee.
REVIEW OF SYSTEMS:
(-) fever
(-) vomiting
(-) behavioral changes
(-) cough/colds
(-) loss of consciousness
(-) numbness
(-) seizures
(-) weakness
(-) urinary/bowel habit changes
(-) abdominal pain
PHYSICAL EXAMINATION:
VITAL SIGNS: HR 96, RR 18, BP 120/80, weight 36 kg.
HEENT: Pink conjunctivae, anicteric sclerae. No nasoaural discharge. No CLAD and dental
caries. No neck mass.
SKIN: No cutaneous lesions.
CHEST/LUNGS: Clear breath sounds. Distinct heart sounds, regular rhythm, no murmur.
ABDOMEN: Flat, noromoactive bowel sounds; no organomegaly.
EXTREMITIES: Full pulses, pink nailbeds, no edema.
NEUROLOGIC EXAMINATION:
MSE: Drowsy, occasionally follows simple commands.
CRANIAL NERVES:
Pupils: 4 mm equal and briskly reactive to light.
Fundosopic Exam: Indistinct disc borders, positive papilledema, OU, no hemorrhage or
exudates.
Left lateral rectus palsy.
Good masseter tone.
No facial asymmetry.
Gross hearing intact.
Equal palatal elevation.
Good SCM tone.
Tongue midline.
MOTORS: Normal muscle bulk and tone, no preferential spontaneous movement.
REFLEXES: +++ on all.
SENSORY: Withdraws to pain.
CEREBELLARS: No dysmetria, no ataxia.
MENINGEALS: Supple neck.
No involuntary movements
Positive bilateral Babinski, no clonus.
CRANIAL CT SCAN: Lobulated hyperdense 6 x 5 x 6.5 cm mass in the right parietal lobe
extending to the thalamus, right temporal lobe and pons. Displacement of the falx to the left
and compression of the 3rd and right lateral ventricle; dilated left lateral ventricle. Diffuse
cerebral edema. Hyperdense lesion 3 x 2 x 1.5 cm between frontal horns of lateral ventricle.
Guide Questions:
1.
2.
3.
4.
5.
What’s the diagnosis? Give the basis and pathophysiology.
What are your differential diagnosis?
What diagnostic test/s would you request for?
What are your therapeutic intervention/s?
Are there preventive measures for this case?
PEDIATRIC CASE NUMBER 7-2
GENERAL DATA: Patient is a 14-year-old female from Taguig and was admitted at PGH on
July 18, 1999.
CHIEF COMPLAINT: Headache.
HISTORY OF PRESENT ILLNESS: Twenty days PTA, patent had moderate to severe
throbbing headache at the occipital area occasionally waking her up from sleep associated with
dizziness and vomiting. Three days PTA, there is progressive headache now involving
bifrontal area with generalized weakness.
PAST MEDICAL HISTORY: Patient has been wearing corrective lenses since 10 years old. No
previous hospitalizations.
BIRTH/MATERNAL/DEVELOPMENTAL HISTORY: Unremarkable.
MENSTRUAL HISTORY: Menarche at 9 years old, regular monthly intervals.
FAMILY HISTORY: Unremarkable.
REVIEW OF SYSTEMS:
(-) seizures
(-) fever
(-) loss of consciousness
(-) ear discharge
(-) blurring of vision
(-) bowel/urinary
PHYSICAL EXAMINATION: BP 130/90, HR 100 per minute, RR 18 per minute, afebrile.
Weight is 37 kg. Unremarkable systemic PE.
NEUROLOGIC EXAMINATION;:
MSE
conscious, coherent, oriented to 3 spheres; with some difficulty with speech
CN II
(+) bilateral papilledema;
Visual Acuity: grossly intact, OU
CN III,IV,VI pupils 3 mm equal, briskly reactive to light
bilateral LR palsy
CN V
brisk corneals
CN VII
(-) facial asymmetry
CN VIII
grossly intact
CN IX,X
equal palatal elevation on phonation
CN XI
equal shoulder shrug
CN XII
tongue midline
MOTORS:
normal tone, (-) atrophy, 5/5
SENSORY:
intact
MSRs:
++, (-) Babinski, (-) clonus
CEREBELLARS:
(+) horizontal nystagmus
(+) dysmetria, left
MENINGEALS:
AUTONOMICS:
(+) dysdiadochokinesia, left
Gait: wide-based
Romberg’s: falls to the left
Tendency to fall to the left on tandem walking
slight nuchal rigidity
(-) Horner’s, abnormal sweat patterns
LABORATORIES:
CBC:
Hgb
Hct
WBC
Segmenters
Lymphocytes 23%
Platelet
URINALYSIS:
RBC
WBC
Bacteria
CRANIAL CT SCAN:
162 g/L
0.46
11.2 x 10/L
66%
293 x 10/L
0-3/hpf
10-15/hpf
+++
Homogenous hyperdense rounded lesion
Left Cerebellar area.
Guide Questions:
6. What’s the diagnosis? Give the basis and pathophysiology.
7. What are your differential diagnosis?
8. What diagnostic test/s would you request for?
9. What are your therapeutic intervention/s?
10. Are there preventive measures for this case?
PEDIATRIC CASE NUMBER 7-3
GENERAL DATA: Patient is a 9-year-old Filipino female. She is right-handed and is a student
from Laguna.
CHIEF COMPLAINT: Headache.
HISTORY OF PRESENT ILLNESS: Six months PTC, patient experienced on and off throbbing
frontal headache on the left occasionally on the right. This was preceded by visual disturbances
described as seeing flashes of light and accompanied by vomiting and photophobia. The
attacks lasted for 1 to 2 hours and relieved by rest. This would occur 1 to 2 times a month. No
consult was done.
A few days PTC, headaches of similar character recurred, becoming more frequent preventing
patient from taking her exam in school. Persistence of symptoms prompted consult.
PAST MEDICAL HISTORY:
hospitalization.
BIRTH MATERNAL HISTORY:
complications.
Negative for trauma or other illness.
No previous
Born FTSVD to a 27-year-old G2P2, (-) fetomaternal
IMMUNIZATION HISTORY: Positive BCG, DPY-OPV3, Measles, Hep B3, and MMR.
NUTRITIONAL HISTORY: Breastfed for 6 months then formula fed thereafter, presently no
food preferences.
DEVELOPMENTAL HISTORY: At par with age.
FAMILY HISTORY: Grandmother is positive for hypertension and mother is positive for
migraines.
PERSONAL/SOCIAL HISTORY: Patient has no vices. She is an average grade 3 student, 2nd
of 2 children to a 36-year-old housewife and a 38-year-old engineer.
REVIEW OF SYSTEMS:
(-) fever
(-) increased sleeping time
(-) behavioral changes
(-) cough/colds
(-) loss of consciousness
(-) numbness
(-) seizures
(-) weakness
(-) urinary/bowel habit changes
(-) abdominal pain
PHYSICAL EXAMINATION:
VITAL SIGNS: HR 90, RR 18, BP 100/60, weight 25 kg.
HEENT: Pink conjunctivae, anicteric sclerae. No nasoaural discharge. No CLAD and dental
caries. No neck mass.
SKIN: No cutaneous lesions.
CHEST/LUNGS: Clear breath sounds. Distinct heart sounds, regular rhythm, no murmur.
ABDOMEN: Flat, noromoactive bowel sounds; no organomegaly.
EXTREMITIES: Full pulses, pink nailbeds, no edema.
NEUROLOGIC EXAMINATION:
MSE: Awake, coherent, oriented to 3 spheres, good memory, calculation fond of information,
good insight, intact judgement.
CRANIAL NERVES
Visual acuity: 20/20, OU.
Pupils: 3 mm equal and briskly reactive to light.
Funduscopic exam: Distinct disc borders, no papilledema, no hemorrhage/exudates.
Full EOMs.
Good masseter tone.
No facial asymmetry.
Gross hearing intact.
Good gag reflex.
Equal palatal elevation.
Tongue midline.
MOTORS: Normal muscle bulk, tone and strength.
SENSORY: No deficits.
CEREBELLARS: No dysmetria, no ataxia, no nystagmus
MENINGEALS: Supple neck.
No involuntary movemements.
No Babinski, no clonus.
Guide Questions:
1.
2.
3.
4.
5.
What’s the diagnosis? Give the basis and pathophysiology.
What are your differential diagnosis?
What diagnostic test/s would you request for?
What are your therapeutic intervention/s?
Are there preventive measures for this case?
PEDIATRIC CASE NUMBER 7-4
GENERAL DATA: The patient is a 12-year-old Filipino female who is right-handed and a
student from Cavite.
CHIEF COMPLAINT: Headache.
HISTORY OF PRESENT ILLNESS: Six months PTC, patient experienced on and off dull ache
at the back of the head initially then becoming diffuse lasting for 1 to 2 hours usually occurring
in the afternoon and relieved by paracetamol intake. No other symptoms were noted. No
consult was done.
One week PTC, while studying for an exam, the headaches of similar character recurred,
becoming more frequent. Persistence of symptoms prompted consult.
PAST MEDICAL HISTORY: Negative for trauma or other illnesses. No previous
hospitalization.
BIRTH/MATERNAL HISTORY: Born FTSVD to a 35-year-old G2P2; (-) fetomaternal
complications.
IMMUNIZATION HISTORY: (+) BCG, DPT3-OPV3, Measles, Hep B3
NUTRITIONAL HISTORY: Patient has been breastfed for 1 year then mixed feeding
thereafter; presently no food preferences.
DEVELOPMENTAL HISTORY: At par with age
FAMILY HISTORY: (+) stroke – grandpa
(+) migraine- mother
PERSONAL/SOCIAL HISTORY: No vices; 1st year high school honor student; eldest of 2
children to a 47-year-old housewife and 48-year-old construction worker.
REVIEW OF SYSTEMS:
(-) fever
(-) increased sleeping time
(-) behavioral changes
(-) cough/colds
(-) loss of consciousness
(-) nausea/vomiting
(-) numbness
(-) seizures
(-) weakness
(-) urinary/bowel habit changes
(-) abdominal pain
PHYSICAL EXAMINATION:
VITAL SIGNS: Heart rate 84, respiratory rate 18, blood pressure 100/70, weight 33 kg.
HEENT: Pink conjunctivae, anicteric sclerae, no nasoaural discharge, no CLAD or dental caries,
no neck mass.
CHEST/LUNGS: Clear breath sounds, distinct heart sounds, regular rhythm, no murmur.
ABDOMEN: Flat, normoactive bowel sounds; no organomegaly
Extremities: full pulses, pink nailbeds, no edema
SKIN: No cutaneous lesions.
NEUROLOGIC EXAMINATION:
MSE: Awake, coherent, oriented to 3 spheres, good memory, calculation, fund of information;
good insight, intact judgment.
CRANIAL NERVERS:
Visual acuity: 20/20, OU
Pupils 4 mm equal and briskly reactive to light
Funduscopic exam: distinct disc borders; no papilledema, no hemorrhage/exudates
Full EOMs
Good masseter tone
No facial asymmetry
Gross hearing intact
Good gag reflex
Equal palatal elevation
Tongue midline
MOTORS: Normal muscle bulk, tone and strength.
SENSORY: No deficits.
CEREBELLARS: No dysmetria, no ataxia, no nystagmus.
MENINGEALS: Supple neck.
No involuntary movements.
No Babinski, no clonus.
SEIZURES
PEDIATRIC CASE NUMBER 8-1
The patient is a 9-year-old boy from Bulacan who consulted at the PGH on May 20, 2006 for the
first time for seizure. A few hours prior to consult, while the patient was asleep, he was heard
moaning with note of facial grimacing, clonic jerking of the right side of the body and versive
head movement to the same side. This lasted for about 2 minutes. Post-ictally, the patient was
drowsy and complained of headache. After 2 hours, he was back to his old self. His PE and
Neuro exam were essentially unremarkable.
Guide Questions:
1. What additional information in the history would you ask for and why?
2. What specific physical findings would you look for and why?
3. What is the pathophysiology of seizure?
4. What is your working diagnosis? Differential diagnoses? Bases?
5. What initial laboratory tests will you request for and why?
6. How will you manage the patient therapeutic-wise:
PEDIATRIC CASE NUMBER 8-2
Patient is an 8-year-old boy from Cagayan who consulted at the PGH for the first time on March
20, 2006 for seizure. He has been complaining of on and off vague left temporal headaches. The
events occurred anytime of the day lasting for 2 to 3 hours. During the last 2 days, he also had
episodes of vomiting. Five days ago, jerking of his right arm was noted during sleep, each event
lasting for 2 to 5 minutes. Yesterday, staring for 15 to 30 seconds was noted 3x prompting
consult at the PGH OPD.
On examination, there was note of pimple-like lesions on patient’s face. Whitish patches were
also noted on his trunk and back. There was no murmur or organomegaly. Neuro exam showed
a cooperative but aphasic child. Funduscopic exam showed abnormal findings. The rest on the
cranial nerve examination was normal, Motor bulk and tone was normal all over. Motor power
was normal save for a mild pronator drift on the right. Sensory was intact. Plantar response was
up going on the right.
Guide Questions:
1. What additional information in the history would you ask for and why?
2. What specific physical findings would you look for and why?
3. What is the pathophysiology of seizure in patients with seizures associated with brain
tumors?
4. How would you determine the area where the mass lesion is based on the characteristics
of the patient’s seizures?
5. What is your working diagnosis? Differential diagnoses? Bases?
6. What initial laboratory tests will you request for and why?
7. How will you manage the patient therapeutic-wise:
PEDIATRIC CASE NUMBER 8-3
A 1-year-old boy consulted at the emergency room for seizures described as focal jerking of the
right hand and leg lasting for 5 minutes. Five days prior to admission, the child had been
having moderate grade fever accompanied by cough and yellowish nasal discharge. There is no
family history of epilepsy. Father had a history of a single seizure episode with fever at age 1.
Physical exam showed a fairly nourished child with labored breathing. HR 120/min RR 40/min
Temp 38.5 C. Auscultation of the chest showed bibasal rales. Otoscopic exam revealed a
draining ear on the left. Head is normocephalic with small open flat anterior fontanel. There are
no cranial nerve deficits seen and there is no preferential movement of extremities. Neck is
supple. Deep tendon reflexes are increased and babinski showed an upward going toe
bilaterally.
Guide Questions:
1. What additional information in the HISTORY would you ask for and why?
2. What findings in the PHYSICAL and NEUROLOGICAL examination would you look
for and why?
3. What are the differential diagnosis for a child presenting with fever and seizure?
4. What diagnostic test/s would you request for and why? What are your expected results?
5. What is the pathophysiology of CNS infection and seizure generation?
a.
Explain how seizures are generated in the presence of CNS infection.
b.
Expain how children develop CNS infection in the presence of infection
elsewhere in the body.
c.
What are the etiologic organisms most commonly identified in children based on
symptomatology and age.
6. What are the appropriate management for this child
7. What is the prognosis for treated and untreated CNS infection
PEDIATRIC CASE NUMBER 8-4
A 2-week-old baby was brought to the emergency room for focal jerking of the right hand and
leg lasting for 5 minutes. The baby was born full term by spontaneous vaginal delivery to a
primigravid mother. He was discharged from the nursery after 2 days. At home the baby was
described to have fair suck and activity consuming only half an ounce of milk for every feeding.
On some occasions he would regurgitate the milk given. There is no history of fever.
Physical exam showed a 3kg baby, not in respiratory distress, fairly active with good cry. Vital
signs: HR 110/min RR 30/min Temp 36.5 C. Head is normocephalic with open sunken anterior
fontanel. Ausculatation of the chest and heart were unremarkable. Abdomen is not distended
with normoactive bowel sounds, skin and buccal mucosa is slightly dry. There are no cranial
nerve deficits seen and there is no preferential movement of extremities. Neck is supple. Deep
tendon reflexes are normal and babinski showed an upward-going toe bilaterally.
Guide Questions:
1. What additional information in the HISTORY would you ask for and why?
2. What findings in the PHYSICAL and NEUROLOGICAL examination would you look
for and why?
3. What are the differential diagnosis for an infant presenting with an acute afebrile
seizure?
4. What diagnostic test/s would you request for and why? What are your expected results?
5. What are your therapeutic intervention/s?
6. What is the prognosis for seizures due to transient metabolic derangements
RASHES
PEDIATRIC CASE NUMBER 9-1
GENERAL DATA: Agnes is a 7-year-old female from Makati who sought consult last October
28, 2005.
CHIEF COMPLAINT: Rashes
HISTORY OF PRESENT ILLNESS: Five days PTC, the patient has moderate grade
undocumented fever associated with dry non-productive cough, runny nose, slightly red eyes,
and mild abdominal pain. No consult was done. The mother gave the patient paracetamol for
the fever. Three days PTC, high-grade fever was noted and documented at 40°C. Mother also
noted appearance of maculopapular rash on the upper lateral parts of the neck. Two days prior
to consult, there was a note of fever at 40°C. The rash progressed to the chest, abdomen and
back. Persistence of symptoms prompted consult.
Guide Questions:
1.
What are the possible differential diagnoses in a patient with rashes?
2.
How is the diagnosis of measles in children made? What are the diagnostic work-us
which you will request for Agnes which could suggest measles?
3.
What is the pathophysiology of Measles infection? What are its complications?
4.
How will you treat the patient?
5.
What are the preventive measures for measles?
FEVER
PEDIATRIC CASE NUMBER 10-1
Allan is 5 years old. He was brought to you for consult because of fever of 5 days duration with
accompanying cough and colds. He later on complained of sore throat with episodes of
vomiting. He was given Paracetamol by his mother with temporary lysis of fever, however,
symptoms persisted with accompanying anorexia and myalgia prompting medical consultation.
On physical examination, you noted that Allan was febrile at 38.8 C. He had hyperemic
tonsillar walls with no exudates. His skin was flushed. His BP was normal for age. A
tourniquet test was done on him which was read to be positive. What is your impression of
Allan’s condition?
Guide Questions:
1. What are the possible differential diagnosis in a patient with an acute febrile illness?
2. What are the diagnostic approaches to a patient with an acute febrile illness?
3. How is a diagnosis of dengue in children made? What are the diagnostic work-ups which
you will request for Allan which could suggest a probable dengue infection?
4. Pathophysiology of dengue infection? Grading?
5. How will you treat Allan for a possible dengue infection?
6. What are the preventive measures to control dengue.
PEDIATRIC CASE NUMBER 10-2
Carmel is 9 years old. She was brought for consultation because of fever of 10 days duration
accompanied by anorexia, body malaise, and headache. She was initially given paracetamol but
her temperature never returned to normal. In the next few days, she developed diarrhea and
crampy abdominal pain and her temperature rose to as high as 40 C. Because of this coupled
with increasing severity of abdominal pain, she was brought to your clinic.
Guide Questions:
1. What are the possible differential diagnosis in a patient with an acute febrile illness?
2. What are the diagnostic approaches to a patient with an acute febrile illness?
3. How is a diagnosis of typhoid fever in children made? What are the diagnostic work-ups
which you will request for Carmel which could suggest typhoid infection? What is the
fever pattern in typhoid?
4. What is the pathophysiology of typhoid infection? What are its complications?
5. How will you treat Carmel ?
6. What are the preventive measures to typhoid infection?
PEDIATRIC CASE NUMBER 10-3
Aya is 6 years old. She has been having cough and colds for 5 days now. She had episodes of
low grade fever for which she is being given Paracetamol with relief. Aya’s appetite decreased
a bit after the onset of her illness, however, she remains to be playful and active. She was
brought by her mom to her pediatrician because she had a similar illness before for which she
was given an oral antibiotic. However ,they were advised by the pediatrician that it was
probably a viral infection, a self-limiting illness for which no antibiotic is warranted.
Guide Questions:
1.
2.
3.
4.
What are the possible differential diagnosis in a patient with an acute febrile illness?
What are the diagnostic approaches to a patient with an acute febrile illness?
What is the pathophysiology of the disease?
How will you treat Aya ?
PEDIATRIC CASE NUMBER 10-4
GENERAL DATA: Patient is a 9-year-old male from Pasig who was admitted for the 1st time
last October 7, 2005.
CHIEF COMPLAINT: Fever.
HISTORY OF PRESENT ILLNESS: Four days PTA, there was note of fever, documented at
39ºC. There was no cough, colds, sore throat, or diarrhea. Patient had headache, characterized
as throbbing, and prominent at the frontal area. Positive for myalgia and retro-orbital pain. No
consults done. Patient self-medicated with paracetamol for fever.
Two hours prior to
admission, there was a persistence of symptoms and epistaxis of approximately 1 teaspoon in
amount. Cold compress on the nasal area was done. Persistence of symptoms prompted
consult.
PAST MEDICAL HISTORY: No previous hospitalizations.
BIRTH MATERNAL HISTORY: Patient was born to a 29-year-old G2P1 (1001) mother via
SVD at a lying- in clinic. Mother had regular prenatal check-ups care of a local health center.
She had no maternal illness during gestation.
IMMUNIZATION HISTORY: Positive BCG, DPT3, OPV3, measles, and MMR.
NUTRITIONAL HISTORY: The patient was never breatfed. He was started on milk formula
(Nan). Solid food was introduced at 6 month of age. Patient has no food allergies. At present,
patient consumes 3 meals and 2 snacks daily. Usual meal consists of 1 cup rice, hotdog,
chicken or fish.
DEVELOPMENTAL HISTORY: At par with age. At present, the patient is a Grade 3 student
at a public elementary school with average scholastic standing.
FAMILY HISTORY: Negative DM, PTB, asthma, hypertension, or similar illness in the family.
PERSONAL/SOCIAL HISTORY: The patient is the younger of 2 siblings. Father is a 40-yearold company driver and his mother is a 38-year-old housewife. His older sister is a 12-year-old
Grade 6 student. Patient is fond of eating fish balls. Patient’s family lives in a 1-bedroom
apartment near Pasig river. There is no regular water supply so they store water in covered
barrels.
REVIEW OF SYSTEMS:
(+) lethargy
(-) ear discharge
(-) hematochezia
(-) constipation
(-) joint pains
(+) anorexia
(-) seizures
(-) constipation
(-) hematemesis
(-) dysuria
(-) frequency
(-) passage of worms
PHYSICAL EXAMINATION:
GENERAL: Patient is awake, lethargic, not in cardiorespiratory distress. Weight is 28 kg and
height is 125 cm.
VITAL SIGNS: BP 90/60, HR 110, RR 28, Temperature 39°C.
HEENT: Pink conjunctivae, anicteric sclerae, negative tonsillopharyngeal congestion, positive
cervical lymphadenopathy.
CHEST/LUNGS: Adynamic precordium, distinct heart sounds, regular rate and rhythm, no
murmurs. Equal chest expansion, clear breath sounds, no rales, no wheezes.
ABDOMEN: Abdomen is soft, non-tender with normoactive bowel sounds.
EXTREMITIES: Pink nailbeds, full pulses, no edema or cyanosis. Absence of rashes but
positive tourniquet test.
GENITALIA: Normal external genitalia.
LABORATORY:
CBC: Hgb 118, hct 37, WBC 4.0, seg 56, lymph 46, platelet 315.
FECALYSIS: Brown, formed, rbc 0, wbc 0, no parasites, negative occult blood.
URINALYSIS: Yellow, turbid, SG 1.016, pH 6.0, rbc 0 to 1, wbc, 0 to 1, bacteria none.
CHEST X-RAY: No significant findings.
Guide Questions::
1. What are the possible differential diagnosis in a patient with an acute febrile illness?
2. What are the diagnostic approaches to a patient with an acute febrile illness?
3. How is a diagnosis of dengue in children made? What are the diagnostic work-ups which
you will request for Allan which could suggest a probable dengue infection?
4. Pathophysiology of dengue infection? Grading?
5. How will you treat Allan for a possible dengue infection?
6. What are the preventive measures to control dengue.
PEDIATRIC CASE NUMBER 10-5
GENERAL DATA: This is a case of J.P., a 6-month-old child from Manila who was admitted
for the first time in this institution due to fever.
HISTORY OF PRESENT ILLNESS: Three days PTA, the patient had intermittent fever. No
consult was done but was given paracetamol which afforded temporary relief. Two days PTA,
there was a noted decrease in feeding and patient cried often. One day PTA, patient had
vomiting episode during feeding time.
PHYSICAL EXAMINATION:
GENERAL: Patient was awake and irritable.
VITAL SIGNS: HR 160/ minute, RR 56/minute, and temperature of 38.9. Height 70 cm, weight
6.2 kg.
HEENT: Anicteric sclerae, pink palpebral conjunctiva. There was no alar flaring, TPC, or
CLAD.
CHEST/LUNGS: Symmetrical chest expansion, clear breath sounds, no rales, wheezes, or
retractions.
HEART: Adynamic precordium, regular cardiac rate and rhythm, no murmur.
ABDOMEN: NABS, soft, no organomegaly, tenderness, or masses.
EXTREMITIES: Full pulses, CRT 2 seconds, no edema.
LABORATORY:
URINALYSIS: sp.gr. 1.021
pH 4
Albumin trace
RBC 10-20
WBC 20-30
(+) granular casts
(-) epithelial cells
bacteria 2+
URINE C/S: 50,000 cfu of Escherichia coli
CBC: hgb 115, hct 0.38, plt 388, seg 0.90, lymph 0.10
Guide Questions:
1. What is the most likely diagnosis?
2. What is the pathogenesis of UTI?
3. Evaluation, Diagnosis, and Treatment of UTI.
4. Essential aspects in History and Physical Examination.
ABDOMINAL
PAIN
PEDIATRIC CASE NUMBER 11-1
GENERAL DATA:
December 9, 2005.
Patient is a 12-year-old male from Sta. Ana who sought consult last
CHIEF COMPLAINT: Abdominal pain.
HISTORY OF PRESENT ILLNESS: Two weeks PTC, fever was noted, documented at 38.5º C
and associated with headache, myalgia, and poor appetite. He had abdominal pain, waxing
and waning in character, which was localized at the epigastric area. He had diarrhea
characterized by loose, watery stools, at 3 episodes per day, non-blood streaked. No consults
were done. Patient self-medicated with paracetamol and Oresol. Three days prior to consult,
there was note of high grade fever, documented at 40º C . Patent was then given paracetamol.
Diarrhea resolved spontaneously; however, the abdominal pain increased in severity. Patient
was positive for constipation, anorexia, and poor activity. Persistence of symptoms prompted
consult.
PAST MEDICAL HISTORY: No previous hospitalizations
BIRTH MATERNAL HISTORY: Patient was born to a 30-year-old G3P2 (2002) mother via
SVD at a lying-in clinic. Mother had regular prenatal check-ups care of a local health center.
Mother was negative for maternal illness during gestation.
IMMUNIZATION HISTORY: Positive BCG, DPT3, OPV3, measles, and MMR.
NUTRITIONAL HISTORY: Patient was breastfed until 1 year of age. Mixed feeding with milk
formula was started at 4 months of age. Solid food was introduced at 6 months of age. At
present, patient consumes 3 meals and 2 snacks daily; however, patient is fond of eating “isaw”
during school breaks.
DEVELOPMENTAL HISTORY: At par with age. At present, the patient is a Grade 6 student
with average scholastic standing.
FAMILY HISTORY: Negative DM, PTB, asthma, hypertension, or similar illness in the family.
PERSONAL/SOCIAL HISTORY: The patient is the youngest of 3 siblings. Father is a 43-yearold accountant and his mother is a 40-year-old secretary. His older brother is 17years old, first
year college student while his older sister is 15 years old, 3rd year high school student. Patient
is fond of eating street foods like “isaw” and fish balls with his friends.
REVIEW OF SYSTEMS:
(+) lethargy
(-) cough
(+) nausea
(-) epistaxis
(-) seizures
(-) colds
(+ vomiting
(-) hematemesis
(-) ear discharge
(-) difficulty of breathing
(-) hematochezia
(-) dysuria
(-) passage of worms
PHYSICAL EXAMINATION:
GENERAL: Patient is awake, lethargic, not in cardiorespiratory distress. Weight is 37 kg and
height is 140 cm.
VITAL SIGNS: BP 100/70, HR 92, RR 28, Temperature 39°C.
HEENT: Eyeballs do not appear sunken, pink conjunctivae, anicteric sclerae, positive dental
caries, negative tonsillopharyngeal congestion, positive cervical lymphadenopathy.
CHEST/LUNGS: Adynamic precordium, distinct heart sounds, regular rate and rhythm, no
murmurs. Equal chest expansion, clear breath sounds, no rales, no wheezes.
ABDOMEN: Abdomen is slightly distended and positive for diffuse tenderness on palpation.
Liver edge 4 cm below the right subcostal margin, splenic edge 2 to 3 cm below the left
subcostal margin. Normoactive bowel sounds.
EXTREMITIES: Pink nailbeds, full pulses, no edema or cyanosis.
GENITALIA: Normal external genitalia.
LABORATORY:
CBC: Hgb 118, hct 37, WBC 4.0, seg 56, lymph 46, platelet 315.
FECALYSIS: Brown, formed, rbc 0, wbc 0, no parasites, negative occult blood.
URINALYSIS: Yellow, turbid, SG 1.016, pH 6.0, rbc 0 to 1, wbc, 0 to 1, bacteria none.
CHEST X-RAY: No significant findings.
JAUNDICE
PEDIATRIC CASE NUMBER 12-1
A 2-month-old female was brought to the OPD because of jaundice. She was born FT,NSD to a
28-year-old primigravid, at home, assisted by a midwife. Perinatal course was uneventful. She
was breastfed with good suck; however, the baby was asleep most of the time and feeds only
every 4 to 6 hours. Her mother proudly says that she is a “good baby” because she rarely cries.
Jaundice was noted on the 4th day of life and has remained to the present, hence the
consultation. The stools were not acholic .
On examination, she was noted to be hypotonic with hoarse cry. Vital signs showed CR: 70 RR:
34 T:37°C. Weight was 3.5 kg and her length was 48cm with head circumference of 38 cm. She
had coarse facies, large open fontanelles, macroglossia. Heart and lung findings were essentially
normal. There was note of globular abdomen with umbilical hernia with normal female external
genitalia.
Laboratory Findings:
• FT4 – 1.4 (nv 11-24 pmol/L)
• TSH - 200 (nv 0.3-3.8mIU/L)
• Total biIirubin: 20 B1: 19 B2: 1
Guide Questions:
1. What other information would you like to elicit from the history? Why ?
2. What other physical findings would you look for? Why?
3. Plot your anthropometric indices and interpret the chart.
4. What is the most likely diagnosis?
5. What is the basis for your diagnosis?
6. What are the possible etiologies of your diagnosis?
7. What diagnostic tests will you request?
8. How will you manage this patient?
9. What is the prognosis of this patient?
10. Is it possible to diagnose the patient’s condition based on PE at birth?
PEDIATRIC CASE NUMBER 12-2
HISTORY: A 10-month-old girl consulted at the emergency room for abdominal distension.
Her mother claims that the history started 2 weeks before admission when she started to have
intermittent febrile episodes associated with reappearance of jaundixe, acholic stools and
gradual abdominal enlargement. The child was diagnosed to have extrahepatic biliary atresia
at 2 months and underwent corrective operation for this. Her jaundice resolved after the
surgery and she has been maintained on the following medications: Phenobarbital,
multivitamins and vitamin K. Presently, she can stand with support.
PHYSICAL EXAMINATION:
GENERAL: She was awake but irritable, with generalized jaundice.
VITAL SIGNS: Weight 8 kg, length 70 cm. Blood pressure 80/40, heart rate 100 per minute,
respiratory rate 24 per minute, and temperature 39C.
HEENT: She had pink conjunctivae, icteric sclerae, no nasal or aural discharge and no palpable
cervical lymphadenopathy.
CHEST/LUNGS: Her lung fields were clear. Heart sounds were distinct, with the apex beat at
the 5th ICS, left midclavicular line.
ABDOMEN: The abdomen was distended with an everted umbilicus, prominent periumbilical
veins. Fluid wave was elicited and the liver was hard and smooth at 4 cm below the right costal
margin, 4 cm below the xiphoid, with the spleen palpable at 7 cm below the left costal margin.
EXTREMITIES: Pulses were full, nail beds pink, no clubbing, but palmar erythema was noted.
There was good muscle tone.
LABORATORY EXAMINATION: Laboratory exam showed hemoglobin 8.0 g/L (normal 1215 g/L); white cell count 15.1 x 109/L (normal 4-11 x 109/L), 70% polymorphs, 28%
lymphocytes; platelet count 100 x 1012/L (normal 150-450 x 1012/L); Prothrombin time
International normalized ratio of 1.8; albumin 18 g/L (normal 35-45 g/L), Na 128 mmol/L
(normal 135-145 mmol/L); K 4.0 mmol/L (normal 3.5-5.5 mmol/L). Hepatobiliary ultrasound
showed liver parenchymal disease.
Guide Questions:
1.
What is the cause of ascites in this patient?
2.
What are the other signs of end stage liver disease in this patient?
3.
What is the most appropriate management for this patient?
4.
Are there any other complications of ascites that may occur in this patient? If so, how
would you manage this?
5.
What advice would you give to the mother as to the long term survival of this patient?
PEDIATRIC CASE NUMBER 12-3
GENERAL DATA: Baby Girl G was delivered full term, 37 weeks by pediatric aging. She
weighed 2650 gm, appropriate for gestational age, and delivered by repeat cesarean section,
cephalic, with an APGAR score of 9, 9. She was born to a 24-year-old G1P0 mother who had
regular pre-natal checkups by a private doctor. The mother denied history of infection during
pregnancy and heredofamilial diseases.
PHYSICAL EXAMINATION: Patient was examined awake and in mild respiratory distress.
She had a HR 156/min, RR 68/min, Temp 36.6 C, with good pulses and capillary refill time.
There was note of mild retractions but with clear breath sounds and good air exchange. There
were no gross abnormalities seen. The rest of the physical examination was unremarkable.
HOSPITAL COURSE: Patient was admitted initially in NICU3 due to respiratory distress. She
was placed on oxygen hood at 100% but was shifted to nasal CPAP 6, FiO2 60% because of
persistent grunting and retractions. The following laboratory examinations were ordered: CBC,
platelet count, blood culture, and chest x-ray (with infiltrates on both lung fields).
The patient was started on Piperacillin-tazobactam and Amikacin. Vitamin K was given and
routine newborn care was done. She was monitored in NICU3 for 3 days until there was
improvement in the clinical status and she was weaned from oxygen support. Breast milk was
fed on 2nd day of life; however, on the 3rd day of life, she was noted to have jaundice up to the
mid-abdomen. Bilirubin levels checked and showed the following results:
Tb – 16 mg/dl
Db – 0.5mg/dl
Ib – 15.5 mg/dl
Baby’ s blood type : A+
Maternal blood type : O+
She was then exposed to continuous phototherapy with appropriate shields. Progression of
jaundice was monitored and bilirubin levels were obtained every 12 hours. Patient remained
with good suck by breastfeeding or cup feeding, with good color and activity. She was moving
bowel regularly and urinating freely.
Bilirubin levels were as follows :
Time
4am
4pm
5am
5pm
6am
TB (mg/dl)
14.4
13.3
12.8
11.54
8.62
DB (mg/dl)
0.12
0.16
0.2
0.15
0.17
IB (mg/dl)
14.3
13.1
12.6
11.4
8.4
On 6th hospital day, phototherapy was discontinued. The patient stayed in the hospital for 9
days. For the 7 days she was treated for pneumonia and monitored for hyperbilirubinemia.
She was discharged improved with no trace of jaundice.
Guide Questions:
1. What is the mechanism of bilirubin formation ?
2. What type of jaundice does the patient have ? How do you differentiate pathologic from
physiologic jaundice ?
3. What is the possible cause of jaundice for this particular case ? What are the differential
diagnosis of unconjugated hyperbilirubinemia ?
4. What laboratory examinations would you request for neonatal jaundice ?
5.
How will you treat this patient with unconjugated hyperbilirubinemia? What are their
complications ?
PALLOR
PEDIATRIC CASE NUMBER 13-1
GENERAL DATA: This is the case of EJA, a 2-year-old female from Laguna who was admitted
for the 1st time due to pallor.
HISTORY OF PRESENT ILLNESS: Two days PTA, the patient had cough & fever, Tmax 38oC.
The patient had no difficulty of breathing and continued to have good appetite and activity. No
medications were taken or consults done.
One day PTA, the mother noted that the patient was pale and had tongue ulcers. She was
brought to a local hospital wherein CBC showed Hgb 37, Hct 0.14, WBC 14.7, pmn 0.47,
lymphocytes 0.50, monocytes 0.03, and platelet 287. Thus, they were advised to go to PGH for
transfusion.
PAST MEDICAL HISTORY: No previous hospitalizations. No known food/drug
hypersensitivity. No measles
BIRTH AND MATERNAL HISTORY: Born to a 23-year-old G4P2 (3001) with no prenatal
check-up. Mother denies any illnesses but 2 siblings died at their neonatal period due to
unknown illness. Patient was delivered preterm, 7 months AOG, via SVD at a lying-in clinic
assisted by a midwife with spontaneous cry and activity, BW was 4 lbs.
NUTRITIONAL HISTORY: Breastfed for 1 month then shifted to modified cows milk until 6
moths of age. Then, bottle-fed with Bear Brand and eats rice without viand.
IMMUNIZATION HISTORY: None.
DEVELOPMENTAL HISTORY: Runs well. Identifies body parts. Feeds self
FAMILY MEDICAL HISTORY: (+) PTB – paternal uncle who lives with them. No HPN/
DM/ Cancer.
PERSONAL AND SOCIAL HISTORY: Mother is a 23-year-old housewife. Father is a
30-year-old tricycle driver with no regular income.
REVIEW OF SYSTEMS:
No headache/ seizures
No epistaxis/gum-bleeding/ hematuria/ melena/easy bruisability
No rashes
No abdominal pain/diarrhea/constipation
No joints pains/ edema
PHYSICAL EXAMINATION:
GENERAL: Patient is awake, conscious, coherent, not in cardiorespiratory distress.
VITAL SIGNS: Wt: 9 kg, wt 80.5 cm, BP 90/60, HR 120, RR 38 Temp 37oC.
SKIN: Warm dry skin, no active dermatoses, pale palms and soles.
HEENT: Pale palpebral conjunctivae, anicteric sclerae. No naso-aural discharge. Pale lips, with
dental caries, slightly hyperemic posterior pharyngeal wall, (+) thrush.
NECK: Supple neck, with multiple non-erythematous, soft, non-tender, movable masses at the
cervical area.
LUNGS: Symmetrical chest expansion, no retractions, equal tactile fremiti, clear breath sounds.
HEART: Adynamic precordium, tachycardic, AB 5th LICS MCL, no murmur.
ABDOMEN: Abdomen is globular, soft, NABS, no masses, no hepatosplenomegaly.
GENITALIA: Grossly female genitalia, no vaginal discharge, no inguinal nodes palpated.
EXTREMITIES: Pulses full and equal, no edema, no cyanosis.
LABORATORY EXAMINATIONS:
CBC: RBC
2.72 x 1012/L
Hb
37 g/L
Hct
0.143
MCV
52.4 fL
MCH
13.5 pg
MCHC
258 g/L
RDW
22.6 %
WBC
15.6 x 109/L
Segmenters 0.33
Lymphocytes 0.55
Monocytes
0.06
Eosinophils 0.00
Atypical Lymph0.03
Platelets
188 x 109/L
RETRICS COUNT: 0.034
PBS: Hypochromia ++++
Microcytosis +++
Slight poikilocytosis
Hypochromic ovalocytes
No toxic granules noted
FECALYSIS WITH OCCULT BLOOD: Negative.
CHEST X-RAY: No significant chest findings.
PEDIATRIC CASE NUMBER 13-2
GENERAL DATA: This is the case of EJ S.P., a 14-year-old nulligravid from Cavite who was
admitted for the 1st time due to pallor.
HISTORY OF THE PRESENT ILLNESS: Three months prior to consult, the patient started to
have prolonged and heavy menses which started on October 19 until December 20, 2005,
consuming 7 pads per day, moderately soaked. Consult was done with a private doctor and
she was prescribed with ferrous sulfate 1 tablet t.i.d. taken for 2 weeks. Patient had no menses
after then.
Six days prior to consult, her menses resumed, with the patient consuming 5-6 napkins per day.
Consult was done at a LHC where the patient was referred to PGH and seen by OB-Gyne.
Transrectal ultrasound revealed an endometrial mass probably hematemetra, multicystic
ovaries. The right ovary measures 2.6x2.4x1.4 and the left ovary measures 2.9x 2.6x1.6. Both
ovaries are converted into multiloculated anechoic cysts.
Menarche:
12 years old
Duration:
5days
Amount:
4pads/day
No dysmenorrhea.
Denies any sexual contact.
PAST MEDICAL HISTORY:
No previous hospitalizations.
hypersensitivity. Chicken pox at 11 years old.
No known food/drug
FAMILY MEDICAL HISTORY: No HPN/DM/cancer.
BIRTH AND MATERNAL HISTORY: Non-contributory.
NUTRITIONAL HISTORY: Presently, prefers processed food, i.e., tocino, longanisa and chips.
She doesn't eat vegetables
IMMUNIZATION HISTORY: (+) BCG, DPT3, OPV3, measles (-) Hepa B3.
DEVELOPMENTAL HISTORY:
performance.
Third year high school student with average academic
PERSONAL AND SOCIAL HISTORY: Mother is a 36-year-old domestic helper, single
mother. The patient lives with her maternal grandparents, a single aunt, and two cousins: 7/M
and 5/F.
REVIEW OF SYSTEMS:
With easy fatigability
With headache and dizziness episodes
No fever/cough/colds
No epistaxis/gum-bleeding/ hematuria/ melena/easy bruisability
No rashes
No joints pains
No abdominal pain/diarrhea/constipation
PHYSICAL EXAMINATION:
GENERAL: Awake, conscious, coherent, and not in cardiorespiratory distress.
VITAL SIGNS: Wt 59 kg, ht 150 cm. BP 110/60, HR 85, RR 20, Temp 36.8C.
SKIN: Warm moist skin, no active dermatoses, pale palms and soles.
HEENT: Pale palpebral conjunctivae, anicteric sclerae. No naso-aural discharge. Pale lips, with
dental caries, no tonsillopharyngeal congestion.
NECK: Supple neck, no cervical lymphadenopathy, thyroid not enlarged.
LUNGS: Symmetrical chest expansion, no retractions, equal tactile fremiti, clear breath sounds.
HEART: Adynamic precordium, NRRR, AB 5th LICS MCL, no murmur.
ABDOMEN: Abdomen globular, soft, NABS, no masses, no hepatosplenomegaly.
GENITALIA: Grossly female genitalia, no vaginal discharge, no inguinal nodes palpated.
EXTREMITIES: Pulses full and equal, no edema, no cyanosis.
LABORATORY EXAMINATION:
CBC: RBC
3.25 x 1012/L
Hb
53 g/L
Hct
0.196
MCV
71.8 fL
MCH
18.5 pg
MCHC
290 g/L
RDW
18.3 %
WBC
6.85 x 109/L
Segmenters 0.655
Lymphocytes 0.285
Monocytes
0.029
Eosinophils 0.031
Platelets
571 x 109/L
RETICS COUNT: 0.026
PBS: hypochromia +++
Microcytosis ++
Slight poikilocytosis
No tocic granules noted
PT:
Control
12.4 secs
Patient
10.4 secs
Activity
100%
INR
1.00
PTT: Control
38.2 secs
Patient
35.6 secs
PEDIATRIC CASE NUMBER 13-3
GENERAL DATA: Roberto, a 1½-year-old boy from Dasmarinas, Cavite, was brought to the
Sick Child Clinic with the chief complaint of abdominal enlargement.
HISTORY OF PRESENT ILLNESS: History revealed that 2 months PTC, he had episodes of
diarrhea, characterized as watery, yellowish, non foul-smelling, mucoid and blood-streaked,
occurring 4 to 5 times per day, which was relieved by intake of chloromycetin as prescribed by
a private physician. He also complained of on and off low-grade fever accompanied with cough
and colds.
One month PTC, the mother noted slight abdominal enlargement followed later by facial and
bipedal edema. Low-grade fever and cough persisted with decreased activity, generalized body
weakness and irritability.
One week PTC, he was noted to be pale and anorectic. Three days PTC, his abdomen was noted
to increase in size. (-) bleeding tendencies (-) urinary disturbance (-) easy bruisability.
PAST MEDICAL HISTORY:
hospitalizations.
Frequent URTI’s, (+) passage of worms, (-) previous
BIRTH AND MATERNAL HISTORY: Born FTVSD at home to a 30-year-old G5P4 (4-0-0-4)
mother. BW 2,200 gms. No perinatal complications. Maternal history was unremarkable.
NUTRITIONAL HISTORY: Breastfed for 3 months only since mother was working, then
bottlefed with Alaska condensed milk at 1:8 dilution 4 to 5 times per day up to 1 year of age,
followed by “am” 8 oz 3 to 4 times per day up to the present. Supplementary feedings started
at 7 months in the form of “lugaw”, soup, but usually eats biscuits. At present, patient would
take only 2 tablespoons of “lugaw” and 8 oz of “am” 3-4 x/day. No vitamin supplementation.
DEVELOPMENTAL HISTORY: Smiled at 2 months. Could hold head at 4 months. Rolled
over at 7 months. Stood with support at 15 months. At present, still cannot talk, stand alone, or
walk.
IMMUNIZATION HISTORY: None.
FAMILY HISTORY: Denies PTB; (-) similar illness in the family.
PERSONAL AND SOCIAL HISTORY: Patient is the youngest of 5 living children. Mother is
a plain housewife and uneducated. Father is a construction worker with no regular job,
finished grade 2.
PHYSICAL EXAMINATION:
GENERAL: P/D, P/N, irritable, not in any form of distress, pale.
VITAL SIGNS: HR 110/min, BP 80/50, RR 30/min , ABW 6.5 kg, T 38o C, AHt 65 cm.
HEENT: Thin, sparse, brown hair; (+) flag sign. Pale conjunctivae, anicteric sclerae, (+) Bitot
spots. (-) Alar flare, (-) auronasal discharge. (+) Pale lips and buccal mucosa, (+) angular
cheilosis.
NECK: (+) Multiple cervical lymphadenopathies, matted, firm, movable.
CHEST/LUNGS: Prominent rib cage, (+) rosary beads, equal chest expansion, harsh breath
sounds.
HEART: (+) Grade 1-2/6 systolic murmur best heard at the apex.
ABDOMEN: Abdominal girth 50 cm; globular with prominent superficial abdominal veins,
liver edge 3-4 cm BRCM, firm, smooth with a blunt edge; (-) splenomegaly, (+) fluid wave,
normoactive bowel sounds. No inguinal lymphadenopathies.
EXTERNAL GENITALIA: Normal-looking.
EXTREMITIES: (+) Non-pitting bipedal edema, (+) flaky dermatitis, full and equal peripheral
pulses.
NEUROLOGIC EXAMINATION: Essentially normal.
Guide Questions:
1. What are the salient features in the history and physical examination?
2. Based on the history and physical examination, what are your differential diagnoses?
LABORATORY EXAMINATION:
CBC: Hb = 60 g/L
Eo .03
Hct = 0.18
St .01
RBC = 2.5 x 1012/L
S .45
9
WBC = 9.0 x 10 /L
L .50
Platelet count = 150 x 109/L Mo .01
RBC morphology = hypochromia ++++. Microcytosis ++++
Reticulocyte count = 20 x 109/L
(-) Toxic granulations
ANC = 4,140
LFTs: Total Protein = 26 g/L
SGPT = 88 uMol/L (N.V. 21-72)
Albumin
= 18 g/L
Alk PO4 = 135 uMol/L (N.V. 38-126)
Globulin
= 8 g/L
DB
= 3.43 uMol/L
IB
= 3.43 uMol/L
FECALYSIS: ++ Trichuris
+ Ascaris Ova 0-1/hpf
CHEST X-RAY: Priimary Koch’s Infection
PPD 5 TU: Negative.
Guide Questions:
1. What are the bases for your primary impression of Iron Deficiency Anemia?
2. Describe the pathophysiology of Iron Deficiency Anemia.
3. What are the laboratory aids in the diagnosis of Iron Deficiency Anemia”?
4. What therapeutic interventions would you recommend?
5. What preventive measures would you advise the patient and the family? How
about in the community?
6. Is Iron Deficiency Anemia preventable?
7. How prevalent is Iron Deficiency Anemia in the Philippine setting?
PEDIATRIC CASE NUMBER 13-4
GENERAL DATA: Anne, a 1-year- and 1-month-old female infant from Cainta, Rizal,
consulted at the clinic of a pediatric hematologist in Manila for follow-up because of anemia.
HISTORY OF PRESENT ILLNESS: She was admitted to a private hospital 22 days PTC as a
referral from a company physician because of thrombocytosis. The physician was considering
Kawasaki Disease.
She had a history of intermittent low to moderate grade fever since 11 days PTA. Paracetamol
afforded relief. Eight days PTA, she was seen by a physician with an impression of a systemic
viral infection. Five days PTA, fever lysed. Another consult was done, this time with the
company physician. CBC revealed Hb 109 g/L, Hct 0.33, RBC 5.27, WBC 14.8, seg .59, lym .34,
st .01, Eo .01, Mo .05, platelet 820; peripheral smear showed hypochromic RBC’s. Co-amoxyclav
was prescribed. A repeat CBC a few hours PTA showed a Hb of 108 g/L, WBC 9.06, platelet
count of 760, and PBS showed hypochromia, anisocytosis, poikilocytosis, and lymphocytosis.
The patient was thus advised admission.
FAMILY HISTORY: The mother and a maternal aunt are anemic. The older sibling, a 3 year
old female, is also anemic.
BIRTH/MATERNAL HISTORY: Born full term to a 27-year-old G2P1 (1001) mother via repeat
Cesearean section at a private hospital with no fetomaternal complications.
IMMUNIZATION HISTORY: She has received complete immunizations.
NUTRITIONAL HISTORY: Breastfed until 3 to 4 months, after which she was shifted to Nan
1, and is currently on Promil !:2 dilution. She consumes a few teaspoonfuls of rice, and mashed
vegetables.
The rest of the history was unremarkable.
PHYSICAL EXAMINATION:
GENERAL: Awake, active, not in cardiorespiratory distress.
VIRAL SIGNS: HR 120, RR 36, T 36.70C, wt 10 kg, ht 76 cm.
SKIN: Good skin turgor, no dermatoses.
HEENT: Pale palpebral conjunctivae, anicteric sclerae, no nasal discharge, pale moist lips,
pinkish pharyngeal wall.
NECK: Supple neck, no cervical lymphadenopathy.
LUNGS: Symmetrical chest expansion, no retractions, clear breath sounds, good air exchange.
HEART: Adynamic precordium, normal rate, regular rhythm, no murmur.
ABDOMEN: Flat, soft abdomen, normoactive bowel sounds, no organomegaly.
GENITALIA: Normal female external genitalia.
EXTREMITIES: Pinkish nailbeds, full equal pulses, no edema/cyanosis.
NEUROLOGIC EXAMINATION: Essentially normal.
Guide Questions:
1. What are the salient features in the history and physical examination?
2. Based on the history and physical examination, what are your differential diagnoses?
LABORATORY EXAMINATION:
CBC: Hb 111 g/L Eo .03
Reticulocyte count 16 x 10-3
Hct. 0.33
St .01
RBC 5.29
S .23
WBC 10.5
L .70
Platelet 751 Mo.03
PBS: (Dr. E. A. Melendres)
RBC’s appear adequate
Hypochromia ++ to +++
Microcytosis ++
Occasional NRBC
WBC ~ 9.5 x 109/L
Ba
Ep
S
L
Mo
.02
.01
.21
.71
.05
No abnormal cell
Increased platelets ~ 900 x 109/L, appearing singly
ANC = 1,995
N.B. Absolute Reticulocyte Count (ARC) = 84.6 x 109/.L
PROTIME:
Pt 12.7 sec
PTT: Pt 43.4 sec
C 11.84 sec
C 29.65 sec
Act 79.45 %
INR 1.08
The patient remained stable, afebrile, and active, and was therefore sent home the next day on
an oral iron preparation (Incremin syrup 30 mg/tsp 1 tsp BID at least 30 min. AC), with advice
on her diet.
On clinic follow-up, the patient was only very slightly pale, with no complaints. Laboratory
examinations revealed the ff.:
CBC: Hb 119
Eo .02
Reticulocyte count 7 x 10-3
Hct .36 St .01
RBC 5.77
S .34
WBC 7.28
L .57
Platelet 314
Mo.06
Peripheral smear reading: (Dr. E. A. Melendres)
RBC’s adequate
Hypochromia ++
Microcytosis ++
WBC ~ 9.0 x 109/L
S
L
Mo
.38
.48.
.14
No abnormal cell
Platelets ~ 600 x 109/L, appearing singly
ANC = 3,420
N.B. Absolute Reticulocyte Count (ARC) = 40.4 x 109/L
Serum Iron 5.9 umol/L (N.V. 6.60-30.40)
Protime: Pt 12.11 sec PTT: Pt 41.80 sec
C 11.84 sec
C 29.70 sec
Act 92.16 %
INR 1.03
Guide Questions:
1. What are the bases for your primary impression of Iron Deficiency Anemia?
2. Describe the pathophysiology of Iron Deficiency Anemia.
3. What are the laboratory aids in the diagnosis of Iron Deficiency Anemia?
4. What therapeutic interventions would you recommend?
5. What preventive measures would you advise the patient and the family? How
about in the community?
6. Is Iron Deficiency Anemia preventable?
7. How prevalent is Iron Deficiency Anemia in the Philippine setting?
PEDIATRIC CASE NUMBER 13-5
GENERAL: Jodelyn, a 15-year- and 9-month-old adolescent female from Las Piňas City was
referred to a pediatric hematologist because of pallor.
HISTORY OF PRESENT ILLNESS: The patient herself noted her pallor 6 months PTC. There
were no accompanying symptoms like jaundice, or hematoma formation. She then consulted a
pediatrician 8 days PTC who did a CBC which revealed: Hb 83 g/L, Hct .25, RBC 2.9, WBC 16.3,
S .81, L .19. She was started on an oral iron preparation (Iberet-500) 1 tab b.i.d. Four days PTC
she consulted another pediatrician who repeated the CBC which revealed: Hb 54 g/L, Hct .17,
WBC 12.4, S .73, L.25, Mo .02, Platelet 467, Retic 118. The patient was prescribed an antibiotic
for mild urinary tract infection and mild pneumonia. She then referred the patient to the
hematologist for evaluation of her anemia.
She has regular menses with moderate flow of 5 days’ duration. Her diet includes rice, pork,
chicken, chips, soft drinks. She is not fond of vegetables and fish. Her family history is
negative for blood disorders. She is an incoming freshman in College (computer science).
PHYSICAL EXAMINATION:
GENERAL: Pale, not jaundiced.
VITAL SIGNS: HR 110/min, wt 52.3 kg, ht 148.5 cm.
HEENT: Pale palpebral conjunctivae, anicteric sclerae. Pale tongue and buccal mucosa.
NECK: No enlarged lymph nodes.
CHEST/LUNGS: (+) end inspiratory wheezes.
HEART: Slightly tachycardic, with a grade 1/6 to 2/6 hemic murmur
ABDOMEN: No organomegaly noted.
EXTREMITIES: Pale nailbeds, no skin hemorrhages.
Guide Questions:
1. What are the salient features in the history and physical examination?
2. Based on the history and physical examination, what are your differential
diagnoses?
LABORATORY EXAMINATION:
CBC: Hb 67
Eo .02
Hct .20
St . 01
RBC 2.6
S .60
WBC 7.6
L .32
Platelets sl.↑
Mo.05
Retic 6 x 10-3
PBS: (Dr. E. A. Melendres)
Markedly decreased RBC’s
Hypochromia +++ to ++++
Microcytosis +++ to ++++
Poikilocytosis ++
WBC ~ 9.0 x 109/L
Eo
St
S
L
Mo
.05
.01
.55
.34
.05
No abnormal cell
Platelets ~ 600 x 109/L
ANC = 5,040
N.B. Absolute Reticulocyte Count (ARC) = 15.6 x 109/L
Serum Iron = 3.8 umol/L (N.V. 6.60 – 30.40)
Guide Questions:
1.
What are the bases for your primary impression of Iron Deficiency Anemia?
2.
Describe the pathophysiology of Iron Deficiency Anemia.
3.
What are the laboratory aids in the diagnosis of Iron Deficiency Anemia?
4.
What therapeutic interventions would you recommend?
5.
What preventive measures would you advise the patient and the family? How
about in the community?
6.
Is Iron Deficiency Anemia preventable?
7.
How prevalent is Iron Deficiency Anemia in the Philippine setting?
PEDIATRIC CASE NUMBER 13-6
GENERAL DATA: Shane, a 7-year- and 3-month-old female child from Sto. Tomas, Batangas,
consulted at the clinic of a pediatric hematologist because of anemia.
HISTORY OF PRESENT ILLNESS: Two weeks PTC the patient developed nonproductive
cough associated with fever (38 to 39oC) temporarily relieved with paracetamol. One week PTC
she was brought to a private MD in Batangas who prescribed Cefaclor for 7 days and Ambroxol
t.i.d.
Three days PTC she consulted a private hospital in Manila, where chest x-ray showed mild
bronchopneumonia. CBC showed Hb 101, Hct .30, RBC 4.27, platelet 573, WBC 13.5, S .70, L .21
Mo .09. She was then started on oral Cefuroxime, as well as oral iron. She was also referred to a
pediatric hematologist for evaluation of her anemia. She also had decreased appetite. No
urinary or bowel changes. No abdominal pains. No joint pains.
PAST HISTORY: Unremarkable.
BIRTH AND MATERNAL HISTORY: Born full term via NSD to a 30-year-old primigravid
mother. No fetomaternal complications.
NUTRITIONAL HISTORY: She was breastfed for 2 weeks, then shifted to S-26 at 1:2 dilution
until age 6 months. At present, she drinks Nido milk, eats rice and viands mostly fried chicken
and spaghetti. Eats junk food.
IMMUNIZATION HISTORY: BCG, DPT 3, OPV 3, Hib 3, Hepatitis B 3.
GROWTH AND DEVELOPMENTAL HISTORY: Feeds self. Toilet trained at 3 years of age.
FAMILY HISTORY: (+) bronchial asthma, mother.
PERSONAL AND SOCIAL HISTORY: The patient is the older of 2 siblings. The younger
sibling, a male, is apparently well. The father, age 29, is a seaman. The mother, age 37, works in
Goldilocks. She is presently a grade 1 student.
PHYSICAL EXAMINATION:
GENERAL: Fairly developed, fairly nourished, pale, conscious, coherent, ambulatory.
VITAL SIGNS: Weight = 21 kg, HR = 120/min, BP = 110/80, Height = 110 cm, RR = 30/min,
T = 38o C.
SKIN: Good skin turgor.
HEENT: Pale palpebral conjunctivae, anicteric sclerae. No naso-aural discharge, slight TP
congestion.
NECK: Supple. (+) few 0.5 cm lymph nodes , anterior and posterior triangles, non-tender.
CHEST/LUNGS: Symmetrical expansion, harsh breath sounds, occasional rhonchi on both
bases. No wheezes.
HEART: Adynamic precordium. Tachycardeic, no murmur.
ABDOMEN: Soft, flat, non-tender. Normoactive bowel sounds.
EXTREMITIES: Slightly pale nailbeds. Full and equal pulses.
NEUROLOGIC EXAMINATION: Essentially normal.
Guide Questions:
1.
What are the salient features in the history and physical examination?
2.
Based on the history and physical examination, what are your differential diagnoses?
LABORATORY EXAMINATION:
CBC: Hb 81 g/L
S .83
Hct .23
L .13
12
RBC 3.24 x 10 /L
Mo .04
WBC 11.1 x 109/L
Platelets adequate
Reticulocyte ct = 5 x 10-3 (16.2 x 109/L)
PBS: Dr. E. A. Melendres)
Decreased RBC’s
Occasional NRBC
Hypochromia +++
Microcytosis +++
WBC  10.0 x 109/L
Ba
St
S
L
.01 .01 .82. .11 .05
+ to ++ toxic granulations
No abnormal cell
Platelets  500 x 109/L
ANC = 8,300
Mo
Serum Iron = 2.5 uMol/L (N.V. 6.6–30.40)
Guide Questions:
1. What are the bases for your primary impression of Iron Deficiency Anemia?
2. Discuss the pathophysiology of Iron Deficiency Anemia.
3. What are the laboratory aids in the diagnosis of Iron Deficiency Anemia?
4. What therapeutic interventions would you recommend?
5. What preventive measures would you advise the patient and the family? How about in
the community?
6. Is Iron Deficiency Anemia preventable?
7. How prevalent is Iron Deficiency Anemia in the Philippine setting?
PEDIATRIC CASE NUMBER 13-7
GENERAL DATA: J.C., a 4-year-old girl from Occidental Mindoro, consulted at the clinic of a
pediatric hematologist because of pallor and jaundice.
HISTORY OF PRESENT ILLNESS: She was apparently well until 3 years PTC, at 1 year of age
when she was noted to be pale and jaundiced. No medical consult was made. Less than a year
PTC, abdominal enlargement was noted with a mass at the left upper quadrant area. Three
days PTC, she consulted a pediatrician in Occidental Mindoro who then referred the patient to
this clinic.
She has never been transfused. There is allegedly no history of fever or bleeding tendencies.
PERSONAL AND SOCIAL HISTORY: The patient is the 4th of 5 siblings. The 2 older siblings,
both female, died of the same condition at age 7 and 4 years, respectively. The 3rd sibling, also a
female, is not affected.
The youngest sibling, a 1-year- and 8-month-old boy consulted at the same time for the same
condition. Therefore, four out of five siblings are affected by the same condition.
The father, age 35, is a farmer, while the mother, age 39, is a public school teacher. Both appear
well.
FAMILY HISTORY: Aside from the other siblings, no other relative on either side has a
similar illness.
PHYSICAL EXAMINATION:
GENERAL: Fairly developed, poorly nourished, markedly pale and sallow, weak-looking,
ambulant, not in distress.
VITAL SIGNS: Weight = 10 kg, HR = 120/min, BP = 90/60, Height = 91 cm, RR = 28/min,
T = 37o C.
SKIN: Skin is pale and sallow.
HEENT: Paper white palpebral conjunctivae, subicteric sclerae, pale buccal mucosa.
NECK: No significant lymph nodes.
CHEST/LUNGS: Symmetrical expansion, clear breath sounds, no rales.
HEART: Tachycardeic, (+) hemic murmur, grade 2/6, heard all over the precordium.
ABDOMEN: (+) Splenomegaly 5 cm below the left costal margin. Liver is not palpable.
EXTREMITIES: Pale nailbeds, no skin hemorrhages.
Guide Questions:
1. What are the salient features in the history and physical examination? Does the
patient fulfill the clinical triad of Hemolytic Anemia?
2. Based on the history and physical examination, what are your differential
diagnoses?
LABORATORY EXAMINATION:
CBC: Hb = 31 g/L
Eo .04
Type B Rh positive
Hct = 0.08
St .01
RBC = 1.34 x 1012/L
S .46
9
WBC = 9.5 x 10 /L
L .47
Platelets adequate
Mo .02
Reticulocyte ct = 22 x 10-3 (29.48 x 109/L)
PBS: (Dr. E. A. Melendres)
+++ Hypochromia
+++ Anisocytosis
++++ Poikilocytosis
(+) Target cells
Severe polychromasia
No abnormal cell
Platelets adequate
Coombs’ Test Direct and Indirect: No agglutination
Osmotic Fragility Test, incubated and unincubated phases (c/o PGH):
Slightly increased resistance to hemolysis
Hemoglobin Electrophoresis (c/o MCM):
Fetal hemoglobin and HbA2 present. No HbA. Presence of HbC or HbE is also a
possibility in combination with HbF.
Guide Questions:
1. What tests constitute the diagnostic work-up for Hemolytic Anemia? Describe the
algorithm of laboratory tests that will help you arrive at the diagnosis.
2. What is your primary impression on the case?
3. Describe the pathophysiology of the particular type of Hemolytic Anemia.
4. What therapeutic interventions would you recommend? What oral supplement should
be given to a patient with Hemolytic Anemia?
5. What is the prognosis of the particular type of Hemolytic Anemia?
6. What are the potential complications of Hemolytic Anemia?
PEDIATRIC CASE NUMBER 13-8
GENERAL DATA: Lemar, a 9-month-old male infant from Los Banos, Laguna, was brought to
the clinic of a pediatric hematologist in Manila because of fever, pallor, and jaundice.
HISTORY OF PRESENT ILLNESS: The baby was noted by the parents to have yellowing of
the sclerae since 1 month of age. He remained active with good suck. At age 7 months, pallor
was noted. No consult was done. A week prior to consultation, the baby was noted to have an
abdominal mass. Three days PTC, Lemar developed on and off low to moderated grade fever.
The parents then consulted a clinic in Calamba where a CBC done confirmed anemia (Hb = 60
g/L). He was subsequently referred to Manila.
There was no history of bleeding tendencies nor exposure to insecticides.
BIRTH AND MATERNAL HISTORY: Lemar was born full term by spontaneous vaginal
delivery to a 26-year-old G2P1 (1-0-0-1) mother. There were no perinatal complications.
DEVELOPMENTAL MILESTONES: Within normal.
NUTRITIONAL HISTORY: He is still breastfed, with introduction of solid foods at age 4
months.
IMMUNIZATION HISTORY: BCG, DPT 3 doses, OPV 3 doses, Measles, and Hepatitis B 3
doses.
FAMILY HISTORY: The father was underwent splenectomy at the age of 8 years, diagnosis
unknown to him. The rest of the family history was unremarkable.
PERSONAL AND SOCIAL HISTORY: The patient is the younger of 2 siblings. The older
sibling is a 6-year-old male, apparently healthy. The father, age 26, is regularly employed in an
oil company, while the mother is a housewife.
PHYSICAL EXAMINATION:
GENERAL: Well developed, well nourished, pale, jaundiced, not in distress.
VITAL SIGNS: Weight = 9.0 kg, HR = 150/min, Length = 73.9 cm , RR = 32/min, HC = 46.5 cm,
T = 37.6oC.
SKIN: Pale, sallow, with tinge of yellow.
HEENT: Normocephalic. Pale conjunctivae, subicteric sclerae. No alar flare. No
tonsillopharyngeal congestion.
NECK: Supple, no lymphadenopathy.
CHEST/LUNGS: Symmetrical expansion, bronchovesicular breath sounds, no rales.
HEART: Tachycardeic, regular rhythm. (+) Grade 1/6 to 2/6 soft systolic murmur at the base.
ABDOMEN: Soft, slightly globular, non-tender. Liver is not palpable. Spleen is palpable 5 cm
below the left costal margin, non-tender.
EXTERNAL GENITALIA: Normal.
EXTREMITIES: Pale nailbeds.
NEUROLOGIC EXAMINATION: Essentially normal.
Guide Questions:
1. What are the salient features in the history and physical examination? Does
the patient fulfill the clinical triad of Hemolytic Anemia?
2. Based on the history and physical examination, what are your differential
diagnoses?
LABORATORY EXAMINATION:
CBC: Hb = 50 g/L
Eo .01
Hct = .13
St .02
RBC = 1.92 x 1012/L
S .20
9
WBC = 16.8 x 10 /L
L .74
Platelet ct = 157 x 109/L
Mo .03
Reticulocyte ct = 62 x 10-3 (119.04 x 109/L)
PBS: (Dr. E. A. Melendres)
Markedly decreased RBC’s
(+) Spherocytes  70 %
(+) Fragments with moderate polychromasia
No abnormal cell
Platelets adequate
Reticulocyte stain: No Heinz bodies. Increased reticulocytes.
Coombs’ Test Direct and Indirect: No agglutination
Osmotic Fragility Test (c/o PGH): Increased fragility of the red cells in both the incubated and
unincubated phases.
Hemoglobin Electrophoresis: Hb A1 = 98 %
Hb A2 = 2 %
Guide Questions:
1.
What tests constitute the diagnostic work-up for Hemolytic Anemia? Describe the
algorithm of laboratory tests that will help you arrive at the diagnosis.
2.
What is your primary impression on the case?
3.
Discuss the pathophysiology of the particular type of Hemolytic Anemia.
4.
What therapeutic interventions would you recommend? What oral supplement
should be given to a patient with Hemolytic Anemia?
5.
What is the prognosis of the particular type of Hemolytic Anemia?
6.
What are the potential complications of Hemolytic Anemia?
PEDIATRIC CASE NUMBER 13-9
GENERAL DATA: Cristopher, a 1½-month-old male infant from Pagsanjan, Laguna, consulted
at the clinic of a pediatric hematologist because of pallor.
HISTORY OF PRESENT ILLNESS: At 1 month of age, the baby was noted to have “yellowish
discoloration” of the eyes and skin which was claimed to have progressed to “paleness” at the
present time. One day prior to consult, the patient developed fever accompanied by colds with
watery nasal discharge. The parents noted the baby to become more pale, thus prompting
consult.
BIRTH AND MATERNAL HISTORY: The patient was born FTVSD to a 17-year-old
apparently healthy primigravid mother, assisted by a midwife at home, with a birth weight of
2.5 kg. Perinatal and immediate postnatal periods were uneventful.
The mother had irregular prenatal check-ups with no intake of multivitamins or any drug; no
infection; no exposure to irradiation.
NUTRITIONAL HISTORY: The baby was not breastfed, and is on cow’s milk formula at 1:2
dilution.
FAMILY HISTORY: (+) Leukemia in a paternal uncle.
PERSONAL AND SOCIAL HISTORY: The father is 21 years old, unemployed. The mother is
a 17-year-old student. The baby is under the care of the paternal grandmother.
The baby was apparently exposed to naphthalene balls which were placed among the baby’s
clothes.
PHYSICAL EXAMINATION:
GENERAL: Fairly developed, fairly nourished, pale, irritable, with good cry, not in distress.
ABW = 3.5 kg
IBW = 3.9 kg
ABL = 54 cm
HC = 38.5 cm
VITAL SIGNS: HR = 187/min, RR = 32/min, T = 38.6o C.
SKIN: With generalized pallor, warm, good turgor.
HEENT: Normocephalic, flat anterior fontanel, pale palpebral conjunctivae, slightly icteric
sclerae, pale buccal and gingival mucosae, no tonsillopharyngeal congestion.
NECK: Supple, no lymphadenopathy.
CHEST/LUNGS: Symmetrical expansion, no retractions, clear breath sounds, no rales.
HEART: Tachycardeic, regular rhythm, (+) grade 2/6 systolic murmur at the apex.
ABDOMEN: Globular, soft, non-tender. Liver palpable 2 cm below the RCM with a smooth,
sharp edge. Spleen is palpable 5 cm below the LCM. Normoactive bowel sounds.
EXTERNAL GENITALIA: Normal.
EXTREMITIES: Pale nailbeds, no edema.
NEUROLOGIC EXAMINATION: Essentially normal.
Guide Questions:
1. What are the salient features in the history and physical examination? Does
the patient fulfill the clinical triad of Hemolytic Anemia?
2. Based on the history and physical examination, what are your differential
diagnoses?
LABORATORY EXAMINATION:
CBC: Hb = 35 g/L
Eo .01
No toxic granulations
Hct = .09
St .03
RBC = 1.42 x 1012/L
S .24
Type O Rh positive
WBC = 18.6 x 109/L
L .70
Platelet ct = 300 x 109/L
Mo .02
-3
Reticulocyte ct = 177 x 10 (251.34 x 109/L)
PBS: (Dr. E. A. Melendres)
Hypochromia + to ++
Microcytosis + to ++
Moderately severe polychromasia with 12 NRBC’s/100 WBC
Few fragments
No blasts
Platelets adequate
Reticulocyte stain: (Dr. E. A. Melendres)
Presence of Heinz bodies in 25 % of the RBC’s.
Coombs’ Test Direct and Indirect: No agglutination
Urine urobilinogen and urine bilirutin: Positive
G6PD assay: Adequate
Osmotic Fragility Test: Normal
Repeat G6PD assay after one month: Inadequate
Guide Questions:
1.
What tests constitute the diagnostic work-up for Hemolytic Anemia? Describe the
algorithm of laboratory tests that will help you arrive at the diagnosis.
2.
What is your primary impression on the case?
3.
Discuss the pathophysiology of the particular type of Hemolytic Anemia.
4.
What therapeutic interventions would you recommend? What oral supplement
should be given to a patient with Hemolytic Anemia?
5.
What is the prognosis of the particular type of Hemolytic Anemia?
6.
What are the potential complications of Hemolytic Anemia?
PEDIATRIC CASE NUMBER 13-10
GENERAL DATA: Miguel Francis, a 2½-year-old male child from Camarines Sur, consulted at
the clinic of a pediatric hematologist in Manila because of anemia.
The patient was referred by 2 pediatricians in Naga City for investigation as to the cause of his
anemia (Hb 91 g/L) noted on routine CBC. He was for pediatric clearance prior to surgery for
undescended testes, bilateral. He was started on oral iron supplementation (Iberet syrup) since
4 months PTC.
PAST HISTORY: No previous hospitalization. Never transfused. No allergy.
NUTRITIONAL HISTORY: Breastfed for 2 months, then shifted to cow’s milk. Presently on
table foods and cow’s milk. Drinks milk from the cup. Eats vegetables and fish.
FAMILY HISTORY: The mother and paternal grandmother are anemic, etiology unknown.
PERSONAL AND SOCIAL HISTORY: The patient is an only child. The father, age 28, is a
nurse. The mother, age 24, is a medical technologist working in Naga City.
PHYSICAL EXAMINATION:
GENERAL: Well developed, well nourished, very active, not in distress.
VITAL SIGNS: Weight = 13.6 kg, HR = 106/min, height = 91 cm , RR = 24/min, T = 37o C.
HEENT: Slightly pale conjunctivae, anicteric sclerae. Pinkish tongue and buccal mucosa. No
TP congestion.
NECK: Supple, no lymphadenopathy.
CHEST/LUNGS: Clear breath sounds, no rales.
HEART: Slightly tachycardeic. (+) grade 1-2/6 hemic murmur best heard at the apex.
ABDOMEN: Soft, flat. No organomegaly. Normoactive bowel sounds.
EXTREMITIES: Pink nailbeds.
EXTERNAL GENITALIA: Undescended testes, bilateral.
NEUROLOGIC EXAMINATION: Essentially normal
Guide Questions:
1.
What are the salient features in the history and physical examination? Does
the patient fulfill the clinical triad of Hemolytic Anemia?
2.
Based on the history and physical examination, what are your differential
diagnoses?
LABORATORY EXAMINATION:
CBC: Hb 84 g/L
Eo .03
Hct .28
St .01
RBC 4.89 x 1012/L
S .50
9
WBC 7.6 x 10 /L
L .45
Platelets adequate
Mo .01
Reticulocyte ct = 40 x 10-3 (195.6 x 109/L)
PBS: (Dr. E. A. Melendres)
Slightly decreased RBC’s
6 NRBC’s/100 WBC
Hypochromia ++
Anisocytosis ++
Poikilocytosis +++
Many fragments
Few target cells
No abnormal cell
Platelets slightly increased
RETICULOCYTE STAIN: (Dr. E. A. Melendres)
Many golf ball inclusions seen
Platelets adequate
Remark: Hemolytic picture probably alpha thalassemia.
Type AB Rh positive
Hemoglobin Electrophoresis c/o MRL, PGH:
Hb A1 = 96.5 %
MCV = 57.6
Hb H = 3.5 %
MCH = 17.9
Interpretation:
Low A2, microcytic, hypochromic red cell morphology consistent with alpha thalassemia
syndrome. The presence of a fast moving component makes the thalassemia a 3 alpha gene
deletion or hemoglobin H.
Guide Questions:
1.
What tests constitute the diagnostic work-up for Hemolytic Anemia? Describe the
algorithm of laboratory tests that will help you arrive at the diagnosis.
2.
What is your primary impression on the case?
3.
Describe the pathophysiology of the particular type of Hemolytic Anemia.
4.
What therapeutic interventions would you recommend? What oral supplement
should be given to a patient with Hemolytic Anemia?
5.
What is the prognosis of the particular type of Hemolytic Anemia?
6.
What are the potential complications of Hemolytic Anemia?
PEDIATRIC CASE NUMBER 13-11
HISTORY: A 5-year-old boy was seen at the emergency room for pallor. Four weeks ago, he
was observed to have progressive pallor and easy fatigability. One week later, he was noted to
have easy bruisability and occasional gum bleeding. He also developed on and off fevers with
temperature up to 38.4 C. Although he was prescribed iron supplementation at a local hospital,
his pallor continued to worsen, thus prompting the emergency room visit.
PHYSICAL EXAMINATION: Physical examination at the emergency room showed a fairly
developed, fairly nourished boy weighing 14 kg. Vital signs showed a HR = 126/min, T = 38.2
C, RR = 32/min, BP = 80/60 mmHg. He was markedly pale with alar flaring. Aside from pale
palpebral conjunctivae and oral mucosa, he has multiple enlarged cervical lymphadenopathy,
firm, non-tender, non-movable around 1.5 to 2 cm in largest diameter. Breath sounds were
vesicular with no rales nor wheezes. He was tachycardic with a gallop noted. Pertinent
abdominal findings include a liver edge at 4 cm BRCM and a spleen at 3 cm BLCM. Multiple
petecchiae and ecchymoses were also noted on the trunk an extremities.
Neurologic examination was essentially unremarkable.
Guide Questions:
1.
What additional information in the history would you ask for and why?
2.
What specific physical findings should you look for and why?
3.
What features in the history and physical examination suggest that a
this patient’s anemia may be due to a malignancy?
4.
What is the pathophysiology of acute leukemia?
5.
What is your working diagnosis? Differential diagnosis? Basis?
6.
What initial laboratory tests would you request and why? What are your
Expected findings?
CBC showed a Hemoglobin = 6g/dl, WBC 40,000 with 20% segmenters,30% lymphocytes, 50%
blasts, platelets = 38,000. Peripheral blood smear showed normochromic, normocytic red
blood cells with leukocytosis and predominance of blasts. Platelets were decreased in number.
Bone marrow aspirate showed a hypercellular marrow with few megakaryocytes and markedly
decreased red cell precursors. Predominantly seen in the smear were small to medium blasts
with inconspicuous nucleoli, scanty blue cytoplasm and occasional clefting of the nucleus.
Immunophenotyping indentified a blast population strongly positive for CD10, CD19, CD22.
Cytogenetic studies still pending.
Blood chemistry and chest X-ray are both normal. CSF cytology showed no abnormal cells.
Guide Questions:
1. What is your working diagnosis now?
2. Identify and prioritize a problem list
3. How would you manage the identified problems in this patients?
PEDIATRIC CASE NUMBER 13-12
GENERAL DATA: L.R., is a 16-year-old female from San Pablo, Laguna who was admitted at
UP-PGH due to pallor
HISTORY OF PRESENT ILLNESS: One year PTA, the patient was noted to be pale associated
with body weakness and epigastric pain with vomiting after intake of food. Consult was done
in a local hospital wherein she was prescribed Maalox, Plasil, Buscopan, and multivitamins.
There was temporary relief of symptoms.
Five months PTA, there was persistence of pallor, which prompted consult to a pediatrician
who ordered CBC which then showed anemia. She was prescribed with iron supplements.
Persistence of symptoms prompted another consult and patient was advised to go to a tertiary
hospital where she was then admitted.
PHYSICAL EXAMINATION:
VITAL SIGNS: Blood pressure 130/90, RR 240, HR 110, BW 35 kg, height 144 cm, BSA 1.18.
HEENT: Normocephalic, (-) periorbital edema, pale conjunctivae, anicteric sclerae.
CHEST/LUNGS: Symmetrical chest expansion, clear breath sounds, no rales/wheeze.
HEART: Adynamic precordium, tachycardic, no mumurs.
ABDOMEN: No abdominal masses.
EXTREMITIES: No pedal edema.
NEUROLOGICAL EXAMINATION: No lateralizing signs.
LABORATORY:
CBC: Hgb 68, hct .21, plt 181, seg 0.66, lymph 0.19, mono 0.10, eos 0.03
ELECTROLYTES:
Na 138 mmol/L
(140-148 mmol/L)
K 3.6 mmol/L
(3.6 – 5.2 mmol/L)
Alb 25 g/L
(34 – 50 g/L)
Phosphorus 1.79 mmol/L
(0.81 – 1.58 mmol/L)
BUN 50 mmol/L
(2.60 – 6.40 mmol/L)
Crea 254 mmol/L
(53 – 115 umol/L)
ABG: pH 7.34
pCO2 35
pO2 83
HCO3 12.5
BE –9.7
sats 97%
URINALYSIS: yellow/hazy
sp.gr. 1.020
sugar negative
pH 6.5
protein 4+
RBC 0-2/hpf
WBC 0-3/hpf
cast negative
Crystals negative
bacteria rare
epithelial cells occasional
Mucus threads negative
KUB ULTRASOUND: Kidneys are small with marked increase in echogenecity and poor
corticomedullary differentiation.
The right kidney measures 8.3 cm x 2.9 cm x 3.0 cm with parynchemal thickness of 0.9 cm.
The left kidney measures 8.2 cm x 3,4 cm x 3.9 cm with parynchemal thickness of 1.0 cm.
The central echo complexes are intact. No lithiasis seen. The urinary bladder is partially filled,
precluding its adequate assessment.
Guide Questions:
1. Define CRF.
2. Define anemia in CRF.
3. What are the causes of anemia in CRF?
4. What diagnostic tools are to be utilized? What is the pathogenesis of the disease?
5. Evaluation and Treatment of patients with anemia due to CRF.
ETC…
PEDIATRIC CASE NUMBER(ENDO)
CONSTIPATION W/ ABDOMINAL PAIN
A 2-year-old male was referred because of constipation accompanied by abdominal pain. His
last bowel movement was 5 days ago. Mother claims that he prefers milk more that solid food.
At present, he only can stand with support, grasp toys, and babbles. Upon observation, his
facial features were not similar to his parents and siblings. Physical examination revealed a
quiet child with a cardiac rate of 80, respiratory rate of 24, afebrile. His weight was 10 kg and
length of 77 cm. He had dry skin, pale palpebral conjunctiva, with still open anterior fontanelle
(3x3 cm), multiple dental carries, macroglossia, low nasal bridge, distended abdomen with
hypoactive bowel sounds and umbilical hernia. Grossly male external genitalia.
Guide Questions:
1. What other information would you like to elicit from the history? Why ?
2. What other physical findings would you look for? Why?
3. Plot your anthropometric indices and interpret the chart.
4. What is the most likely diagnosis?
5. What is the basis for your diagnosis?
6. What are the possible etiologies of your diagnosis?
7. What diagnostic tests will you request?
8. How will you manage this patient?
9. What is the prognosis of this patient?
10. Is it possible to diagnose the patient’s condition based on PE at birth?
PEDIATRIC CASE NUMBER(NEURO)
POOR FEEDING
GENERAL DATA: A 5-day-old baby girl was brought to the ER for the first time.
CHIEF COMPLAINT: Poor feeding.
HISTORY OF PRESENT ILLNESS: Patient was born full term via spontaneous vaginal
delivery at local hospital to 25-year-old G2P2 with no complications. Patient was discharged the
following day with good suck and activity.
On the 4th day of life, there was note of decreased feeding. Persistence of this
symptom prompted consult.
NUTRITIONAL HISTORY: Breastfed since birth.
IMMUNIZATION HISTORY: None.
PERSONAL/SOCIAL HISTORY: Mother is a 25-year-old housewife and father is a 30-year-old
factory worker.
FAMILY HISTORY: No similar illness; no seizure disorder.
REVIEW OF SYSTEMS:
(-) fever, cough, colds, diarrhea, seizures
(+) non projectile vomiting 1 episode
PHYSICAL EXAMINATION:
VIRAL SIGNS: HR 140
RR 36 T 37°C
ABW 3 kg
ABL 50 cm
HC 35 cm
CC 33 cm
AC 33 cm
CBG 40 mg/dl
Open full soft fontanels
No dysmorphic features
HEENT: Unremarkable.
HEART: Regular rhythm, no murmur.
LUNGS: Clear breath sounds.
ABDOMEN: NABS, no hepatosplenomegaly.
EXTREMITIES: Full pulses, pink nailbeds, no deformities.
SKIN: No lesions.
NEUROLOGIC EXAMINATION:
Spontaneous eye opening but drifts easily to sleep.
CRANIAL NERVES: 3 mm EBRTL, (+) Doll’s and corneal reflex .
No facial asymmetry, tongue: no atrophy.
MOTOR: Normal muscle tone; no preferential movement.
SENSORY: Withdraws to pain.
DEEP TENDON REFLEXES: Normal reflexes.
No nystagmus
Bilateral Babinski, unsustained clonus
Supple neck
COURSE IN THE HOSPITAL:
On 1st HD, patient developed generalized clonic seizures, opisthotonos and episodes of
apnea. She then became stuporous.
CBC: Normal
Urinalysis: unusual odor of urine and (+) ketones; rest unremarkable
ABG – metabolic acidosis
Septic work up – negative
Ammonia – normal levels
NBS: negative
UMS: (+) to dinitrophenylhydrazine (DNPH)
Plasma amino acid analysis: elevated leucine, isoleucine and valine
PEDIATRIC CASE NUMBER (ENDO)
CHOLESTEROLEMIA
A 1-year- and 4-month-old female was referred because of elevated blood cholesterol level. She
had unremarkable birth history. She was observed to be a quiet baby. At 2 months old, bowel
movement was noted once every 2 to 3 days with poor activity. At 6 months old, she was noted
to look different from her parents and was noted to have feeding difficulty. She was brought to
a pediatrician and impression was congenital hypothyroidism.
Thyroid function test was requested. Specimen serum for the test was noted to be lipemic. Lipid
function revealed elevated cholesterol level. HR-110/min RR-25/min T-37’C, Wt- 7.8kg Ht69cm HC- 48cm. She had anicteric sclerae, pink palpebral conjunctivae, wide open ant
fontanelles, (+) flat nasal bridge, (+) macroglossia, (+) no tooth eruption, clear breath sounds,
doughy consistency of extremities, grossly female genitalia.
Laboratory Findings:
• FT4 – 2.4 (nv 11-24 pmol/L)
• TSH - >100 (nv 0.3-3.8mIU/L)
• Total cholesterol – 1200 (nv <200 mg/dl)
• Greulich Pyle: standard female of <1y/o
• CBC: 88.4/0.26/6.5/0.17/0.83/265
CXR – normal
Guide Questions:
1. What other information would you like to elicit from the history? Why ?
2. What other physical findings would you look for? Why?
3. Plot your anthropometric indices and interpret the chart.
4. What is the most likely diagnosis?
5. What is the basis for your diagnosis?
6. What are the possible etiologies of your diagnosis?
7. What diagnostic tests will you request?
8. How will you manage this patient?
9. What is the prognosis of this patient?
10. Is it possible to diagnose the patient’s condition based on PE at birth?